You are on page 1of 133

1

MISSION 35+
(EXPECTED CURRENT AFFAIRS QUESTIONS IN CLAT)
.. we covered 13 of the 14 passages in the last CLATs, we intend to cover all 7 this time as well ☺
https://t.me/divyakumargarg

Vol. I (2023)
August 9th 2022

2
Food Inflation 6
INS Surat and frigate INS Udaygiri 7
HS200 Solid Rocket Booster for Gaganyaan Mission 9
Dollarisation due to Cryptocurrencies 11

Twin Cyclones 13
Indo-Pacific Economic Framework 14
Trincomalee Port of Sri Lanka 16
Gun Control Regulation 18
US-Taiwan Relations 19

7th Annual Meeting of NDB 22


Stagflation 24
Pradhan Mantri Awas Yojana 25
India and Gulf Countries 27
'Tomb of Sand' won International Booker Prize 29

Reservation in Promotion 31
Bilateral Security and Defense Cooperation: Indo-Japan 33
UN-Habitat Plan for Jaipur 35
Cultural Significance of PM’s Gifts for QUAD Leaders 37
Bharat Drone Mahotsav 2022 39

India Vietnam Partnership 41


QS World University Ranking 42
Israel Signs Free Trade Deal with UAE 44
Thailand makes Marijuana Legal 46
India's Trade with the USA 48
China’s Tiangong Space Station 50

Agneepath Scheme 51

3
Sovereign Gold Bond Scheme 53
24th ASEAN India Meeting 55
8th International Day of Yoga 56

WEB 5.0 58
Aspirational District Programme 59
India Patent Regime Clash with the U. S. norms 61
Jal Jeevan Mission 63
14th BRICS Summit 65
One Candidate One Constituency 67
Governor’s Power to Call for a Floor Test 68
Stockholm+50 70
Goods and Services Tax Council 72
48th G7 Summit 74

SC Judgment on Eco Sensitive Zone 76


https://unacademy.com/course/mission-35-g7-summit-eco-sensitive-zone/TN3BNQNE 76
Transfer of in-orbit Communication to NewSpace India Ltd (NSIL) 77
UN Report on the Taliban Regime 79
India Tajikistan Bilateral Relations 81

SHRESHTA Scheme 83
China’s Presence in the Horn of Africa 0 85
India's First Liquid Mirror Telescope 87
New POEM Platform 89
Ban on Export of Sugar 91
National Investigation Agency 92

75th Session of World Health Assembly 94


Coalition for Disaster Resilient Infrastructure (CDRI) 96
Australia India Water Security Initiative 98

4
Election to the Office of Vice President in India 100
China’s new High-Tech Aircraft Carrier Fujian 102
Indian Railways Innovation Policy 103

I2U2 Summit and Food Security 105


Issue with the Juvenile Justice Amendment Act, 2021 107
National Initiative for Promoting Upskilling of Nirman Workers (NIPUN) 109
New VPN Rules 111
CAATSA 113
India-Australia Critical Minerals Investment Partnership 115
NIRF Rankings 2022 117
World Population Prospects 2022 119
The African Union at 20 120
Sri Lanka Crisis 122

Registered Unrecognized Political Parties (RUPP) 124


National Legal Services Authority 127
Monkeypox 128
Beti Bachao Beti Padhao Scheme 130

5
Food Inflation

https://unacademy.com/course/mission-35-food-inflation-ins-surat-and-udaygiri/VKWJ5LE1

1. Russia and Ukraine supply what percentage of global wheat exports?

a. about 40%

b. about 65%

c. about 30%

d. about 45%

2. Which organization publishes the World food price index every year?

a. World Food Fund

b. UNICEF

c. Amnesty International

d. UN Food and Agriculture Organization

3. Consider the following statements regarding how Global Prices of Food Affect Domestic Prices.

I. The transmission of the above global inflation to domestic food prices basically depends on how
much of a country’s consumption/production is imported/exported.

II. Such transmission is evident in edible oils and cotton, where up to two-thirds of India’s
consumption and a fifth of its production are imported and exported, respectively.

Which of the above statements is correct?

a. Only I

b. Only II

6
c. Both I & II

d. Neither I nor II

4. What is the rate at which the government purchases crops from farmers, and is based on a
calculation of at least one-and-a-half times the cost of production incurred by the farmers?

a. MSP

b. MRP

c. MCP

d. None of the above

5. In light of the Russian-Ukrainian conflict, Russia captured Crimea in which year?

a. 2013

b. 2014

c. 2015

d. 2016

1. c. about 30%.

2. d. UN Food and Agriculture Organization

3. c. Both I & II

4. a. MSP.

5. b. 2014

INS Surat and frigate INS Udaygiri

https://unacademy.com/course/mission-35-food-inflation-ins-surat-and-udaygiri/VKWJ5LE1

7
1. The recent frigate INS ship 'Udaygiri' is named after a mountain range in which of the following
states?
a. Tamil Nadu
b. Andhra Pradesh
c. Karnataka
d. Kerala

2. The recently launched INS Surat’ is the _______ ship of Project 15B Destroyers which heralds
a significant makeover of the P15A (Kolkata Class) Destroyers.
a. Second
b. Third
c. Fourth
d. Fifth

3. Consider the following statements in relation to the significance of the Indian Navy's Project
75.
I. It is a programme by the Indian Navy that entails building two Scorpene-Class attack
submarines.
II. The programme has been undertaken with the transfer of technology from the French company
Naval Group (formerly known as DCNS) at Mazagon Dock Limited (MDL).
Which of the above statements is/are correct?
a. Only I
b. Only II
c. Both I and II
d. Neither I and II

4. Consider the following statements in relation to the significance of the BRAHMOS Missile.
I. BRAHMOS is a joint venture between the Defense Research and Development Organization of
India (DRDO) and the NPOM of Ukraine.

8
II. It is a two-stage (solid propellant engine in the first stage and liquid ramjet in the second) air to
surface missile with a flight range of around 300 km.
Which of the above statements is/are correct?
a. Only I
b. Only II
c. Both I and II
d. Neither I and II

5. The BRAHMOS missile is named after the Brahmaputra river and ________river in Russia.
a. Muskova
b. Moskva
c. Monsova
d. Moksona

1. b. Andhra Pradesh
2. c. Fourth
3. b. Only II
4. Only II
5. b. Moskva

HS200 Solid Rocket Booster for Gaganyaan Mission

https://unacademy.com/course/mission-35-gaganyan-mission-dollarisation/U13URU6E

1. Which of the following GSLVs will become part of the Gaganyaan Mission to carry Indian
astronauts to space?
a. GSLV Mk I
b. GSLV Mk II
c. GSLV Mk III
d. GSLV Mk IV

9
2. Consider the following statements in relation to the significance of the Indian Space Research
Organization (ISRO).
I. ISRO is the space agency under the Department of Space of the Government of India,
headquartered in the city of Bengaluru, Karnataka.
II. Antrix Corporation Limited (ACL) is a Marketing arm of ISRO for the promotion and
commercial exploitation of space products, technical consultancy services and transfer of
technologies developed by ISRO.
Which of the above statements is/are incorrect?
a. Only I
b. Only II
c. Both I and II
d. Neither I and II

3. Consider the following statements in relation to the functioning of the GSLV Mk III.
I. GSLV MkIII, chosen to launch the Chandrayaan-2 spacecraft, is a four-stage heavy-lift launch
vehicle developed by ISRO.
II. GSLV Mk III is designed to carry a 4 ton class of satellites into Geosynchronous Transfer Orbit
(GTO) or about 10 tons to Low Earth Orbit (LEO), which is about twice the capability of the
GSLV Mk II.
Which of the above statements is/are incorrect?
a. Only I
b. Only II
c. Both I and II
d. Neither I and II

4. India's Gaganyaan Mission has been planned to be launched by which year?


a. 2022
b. 2023
c. 2024
d. 2025

10
5. The Gaganyaan system module, called the Orbital Module will have ______ Indian astronauts,
including a woman.
a. Two
b. Three
c. Four
d. Five

1. c. GSLV Mk III
2. d. Neither I and II
3. a. Only I
4. b. 2023
5. b. Three

Dollarisation due to Cryptocurrencies

https://unacademy.com/course/mission-35-gaganyan-mission-dollarisation/U13URU6E

1. _________is a form of currency substitution, where dollars are used in addition to or instead of
the local currency of a country.
a. Dollarisation
b. Re-dollarisation
c. De-dollarisation
d. None of the above

2. Consider the following statements in relation to the significance of the Reserve Bank of India
(RBI).
I. The Reserve Bank of India was established on April 1, 1935, in accordance with the provisions
of the Reserve Bank of India Act, 1934.
II. Though originally privately owned, since its nationalization in 1945, the Reserve Bank is fully
owned by the Government of India.
Which of the above statements is/are correct?

11
a. Only I
b. Only II
c. Both I and II
d. Neither I and II

4. Consider the following statements in relation to the regulation of cryptocurrencies in India.


I. In 2018, the RBI issued a circular preventing all banks from dealing in cryptocurrencies which
were declared unconstitutional by the Supreme Court in May 2020.
II. Recently, the government has announced to introduce a bill; Cryptocurrency and Regulation of
Official Digital Currency Bill, 2021, to create a sovereign digital currency and simultaneously ban
all public cryptocurrencies.
Which of the above statements is/are correct?
a. Only I
b. Only II
c. Both I and II
d. Neither I and II

5. In which city, the Central Office of Reserve Bank of India was initially established?
a. Calcutta
b. Mumbai
c. New Delhi
d. Madras

1. a. Dollarisation
2. a. Only I
3. a. Only I
4. a. Calcutta

12
Twin Cyclones

https://unacademy.com/course/mission-35-twin-cyclone-indo-pacific-economic-
framework/WHG1ZT6C

1. Recently, twin cyclones named Asani and ____ have been captured in satellite images in the
Indian Ocean region.
a. Karnil
b. Karim
c. Krit
d. Karna

2. Twin tropical cyclones are caused by which of the following types of waves?
a. Rossby waves
b. Topical waves
c. Traverse waves
d. Longitudinal waves

3. The twin cyclone name Asani has been suggested by which of the following countries?
a. Sri Lanka
b. Myanmar
c. Bhutan
d. Bangladesh

4. Consider the following statements in relation to the significance of the Indian Ocean Region.
I. India’s SAGAR policy is an integrated regional framework, unveiled by the Indian Prime
Minister during a visit to Mauritius in March 2015.
II. The primary responsibility for peace, stability and prosperity in the IOR would be on those
“who live in this region”.
Which of the above statements is/are incorrect?

13
a. Only I
b. Only II
c. Both I and II
d. Neither I and II

5. Consider the following statements in relation to the Hurricanes.


I. A hurricane starts out as a tropical disturbance in an area over warm ocean waters where rain
clouds are building.
II. In Australia, hurricanes are also known as "typhoons".
Which of the above statements is/are incorrect?
a. Only I
b. Only II
c. Both I and II
d. Neither I and II
1. b. Karim
2. a. Rossby waves
3. a. Sri Lanka
4. d. Neither I and II
5. b. Only II

Indo-Pacific Economic Framework

https://unacademy.com/course/mission-35-twin-cyclone-indo-pacific-economic-
framework/WHG1ZT6C

1. The second in-person summit of the Quad leaders was held in which city?

a. Tokyo

b. Chennai

c. Sydney

14
d. Seoul

2. Which of the following countries is not a member of the Quad?

a, S. Korea

b. India

c. japan

d. USA

3. AUKUS is a security alliance between Australia, US, and which of the following countries?

a, Japan

b. UK

c. India

d. S. Korea

4. India has reiterated her commitment to the Paris Climate Accord, prior to the Climate Ambition
Summit which will start from 12th December 2020 at which of the following places?

a, Norway

b. Sweden

c. Scotland

d. Wales

5. The QUAD was formed in which year?

a, 2007

b. 2017

15
c. 2019

d. 2020

1. a. Tokyo.
2. a. S. Korea.
3. b. UK.
4. c. Scotland.
5. b. 2017.

Trincomalee Port of Sri Lanka

https://unacademy.com/course/mission-35-trincomalee-port-gun-control/2C9DHIK8

1. Recently, which port situated in Sri Lanka is being planned to develop into an industrial hub for
sparking local and foreign investment?
A. Hambantota Port
B. Trincomalee Port
C. Colombo Port
D. Port of Gallee

2. Trincomalee Port in SL, which is being developed as a global industrial hub, is nearest to the
following cities in India?
A. Pondicherry
B. Chennai
C. Madurai
D. Kozhikode

3. Which of the following countries has commissioned the 2020 ADB (Asian Development Bank)
study on the development of Trincomalee port?
A. China
B. India

16
C. Nepal
D. Japan

4. Consider the following statements in relation to the functioning of the Asian Development Bank
(ADB).
I. ADB is a regional development bank established in 1966 which aims to promote social and
economic development in Asia and the Pacific.
II. It is headquartered in Geneva, Switzerland.
Which of the above statements is/are incorrect?
A.. Only I
B. Only II
C. Both I and II
D. Neither I and II

5. Consider the following statements in relation to the significance of Hambantota Port in Sri
Lanka.
I. China has formal control over Sri Lanka’s Hambantota port as a part of a 99-year lease.
II. Leasing of Hambantota and the Colombo Port City project makes it almost certain for the
Chinese navy to have a permanent presence in the Indian Ocean, also known as China's Strings of
Pearl Strategy.
Which of the above statements is/are incorrect?
A.. Only I
B. Only II
C. Both I and II
D. Neither I and II

Answers:
1. B
2. B
3. D
4. B

17
5. D

Gun Control Regulation

https://unacademy.com/course/mission-35-trincomalee-port-gun-control/2C9DHIK8

1. Consider the following statements in relation to the requirements of acquiring a gun license in
India.
I. The applicant must not have been convicted of any offense involving violence or moral turpitude
five years prior to commencing the application, not of an ‘unsound mind’ and not a threat to public
safety and peace.
II. Property qualification is not a criterion for acquiring a gun license.
Which of the above statements is/are correct?
A.. Only I
B. Only II
C. Both I and II
D. Neither I and II

2. Recently, the United States Supreme Court held that the ________amendment of the US
Constitution protects the right to “keep and bear arms” for self-defense.
A. First
B. Second
C. Third
D. Fourth

3. Which of the following Acts cover all aspects relating to the acquisition, possession,
manufacture, sale, import, export, and transport of arms and ammunition in India?
A. The Arms Act, 1956
B. The Arms Act, 1957
C. The Arms Act, 1958

18
D. The Arms Act, 1959

4. Consider the following statements in relation to the significance of the Arms Act (Amendment),
2019.
I. The Arms Act amended in 2019 reduces the number of firearms that an individual can procure
from 4 to 2.
II. The validity of the license has been increased from the present 3 years to 5 years.
Which of the above statements is/are incorrect?
A.. Only I
B. Only II
C. Both I and II
D. Neither I and II

5. What is the minimum age requirement for acquiring a gun license in India?
A. 20
B. 21
C. 22
D. 24

Answer:
1. C
2. B
3. D
4. A
5. B

US-Taiwan Relations

https://unacademy.com/course/mission-35-us-taiwan-relations-ndb/YMNZ6DPY

19
1.What is the U.S. “One China” policy?

(a) It is the diplomatic acknowledgement of China's position that there is only one Chinese
government.

(b) Under the policy, the US recognises and has formal ties with Taiwan rather than with China.

(c) The One China policy refers to a United States policy which recognizes the ROC as the sole
legal government of China.

(d) None of the Above.

2.Which of the following statements is/are true?

I.The Shanghai Communique (1972), the Normalization Communique (1979) and the 1982
Communique are the three documents outlining the US-China mutual understanding on the Taiwan
question.

II.As per the 1979 communique, the US accepts the ‘one China principle’ considering Taiwan, a
part of China.

(a) I only

(b) II only

(c) Both I and II

(d) Neither I nor II

3.Which of the following statements is/are true?

I. Taiwan is an island territory across the Taiwan Strait, located off the coast of South China Sea.

II.The ruling Kuomintang (Nationalist) government of China fled to Taiwan after being defeated
by the communist forces in the Chinese civil war of 1949-1959.

(a) I only

(b) II only

20
(c) Both I and II

(d) Neither I nor II

4.What is the Taiwan Relations Act, 1979?

(a) Policy of the United States to preserve and promote extensive, close, and friendly commercial,
cultural, and other relations between the people of the United States and China.

(b) Policy of the United States to preserve and promote extensive, close, and friendly commercial,
cultural, and other relations between the people of the United States and the people on Taiwan, as
well as the people on the China mainland.

(c) United States policy which recognizes the ROC as the sole legal government of China.

(d) None of the Above.

5.Which country is related to the Sunflower Student Movement?

(a) Taiwan

(b) China

(c) Japan

(d) USA

1.Answer:(a)

2.Answer:(c) Both I and II

3.Answer:(d) Neither I nor II

4.Answer:(b)

5.Answer:(a) Taiwan

21
7th Annual Meeting of NDB

https://unacademy.com/course/mission-35-us-taiwan-relations-ndb/YMNZ6DPY

1.Where is the headquarters of the New Development bank?

(a) Shanghai, China

(b) Jakarta, Indonesia

(c) Tokyo, Japan

(d) The Hague, Netherlands

2.Which organization founded the New Development bank?

(a) UN

(b) BRICS

(c) ASEAN

(d) APEC

3.Which of the following statements is/are true about New Development bank?

I. In 2018, the NDB received observer status in the United Nations General Assembly.

II.It has committed funding to a number of major infrastructure projects in India, including the
Delhi Metro rail.

(a) I only

(b) II only

(c) Both I and II

(d) Neither I nor II

4.Which of the following statements is/are true about the New Development Bank?

22
I.Its ownership structure is unique, as the BRICS countries have an equal share and no country has
any veto power.

II.All members of the United Nations could be members of the Bank, however the share of the
BRICS nations can never be less than 55% of voting power.

(a) I only

(b) II only

(c) Both I and II

(d) Neither I nor II

5.What is the theme of the 7th Annual Meeting of the New Development Bank?

(a) NDB: Optimizing Banking Sector.

(b) NDB: Optimizing Development Impact.

(c) NDB: Optimizing Banking Impact.

(d) None of the Above.

1.Answer:(a) Shanghai, China

2.Answer:(b) BRICS

3.Answer:(a) I only

4.Answer:(c) Both I and II

5.Answer:(b)

23
Stagflation

https://unacademy.com/course/discussion-on-mission-35-stagflation-pm-aawas-
yojana/41U81T6B

1. Consider the following statements:


I. Stagflation means a situation characterized by simultaneous decrease in prices and stagnation of
economic growth.
II. It is described as a situation in the economy where the growth rate slows down, the level of
unemployment remains steadily high and yet the inflation or price level remains high at the same
time.
Choose the correct option:
a. I only
b. II only
c. Both I and II
d. Neither I nor II

2. The term Stagflation was coined by:


a. John Andraos
b. Stephen J. Benkovic
c. Lain Macleod
d. Robert Welch

3. In 1973, the Organization of Petroleum Exporting Countries (OPEC) decided to cut oil supply.
This led to oil prices soaring across the world, ultimately leading to a situation of Stagflation.
Which among the following is not a member of OPEC?
a. Nigeria
b. Angola
c. Venezuela
d. Qatar

4. Consider the following statements:

24
I. Stagflation can also be defined as a period of inflation combined with a decline in the gross
domestic product (GDP).
II. Generally, stagflation occurs when the money supply is expanding while supply is being
constrained.
Choose the correct option:
a. I only
b. II only
c. Both I and II
d. Neither I nor II

5. Which among the following is not an instrument of Monetary Policy?


a. Repo Rate
b. Open Market Operations
c. Public Debt
d. Bank Rate

1. Answer: B. II only
2. C. Lain Macleod.
3. D. Qatar.
4. Answer: C. Both I and II.
5. C. Public Debt.

Pradhan Mantri Awas Yojana

https://unacademy.com/course/discussion-on-mission-35-stagflation-pm-aawas-
yojana/41U81T6B

1. Consider the following statements:


I. PMAY-G was launched in 2016 with the objective of “Housing for everyone”.
II. It was launched with an aim to provide a pucca house with basic amenities to all rural families,
who are homeless or living in kutcha or dilapidated houses.

25
Choose the correct option:

a. I only

b. II only

c. Both I and II

d. Neither I nor II

2. PMAY-U comes under the ambit of which ministry?


a. Ministry of Rural Development

b. Ministry of Housing and Urban Affairs

c. Ministry of Women and Child Development

d. Ministry of Home Affairs

3. PMAY-U was launched in which year?

a. 2014

b. 2015

c. 2016

d. 2017

4. Consider the following statements:

I. The Pradhan Mantri Awas Yojana-Gramin (PMAY-G) has a completion rate of 67.72%, whereas
the Pradhan Mantri Awas Yojana-Urban (PMAY-U) scheme that started a year ahead is lagging
behind with a 50% completion rate.

26
II. As per the scheme, the Center pays 50% and the States have to bear the rest 50% of the cost.
Choose the correct option:
a. I only
b. II only
c. Both I and II
d. Neither I nor II

5. Which among the following states have a completion rate which is above the national average?
a. Madhya Pradesh and Maharashtra
b. Uttar Pradesh and Bihar
c. Uttar Pradesh and West Bengal
d. Bihar and West Bengal

1. Answer: B. II only
2. B. Ministry of Housing and Urban Affairs.
3. B. 2015.
4. A. I only.
5. C. Uttar Pradesh and West Bengal.

India and Gulf Countries

https://unacademy.com/course/mission-35-india-gulf-countries-international-booker/3U5KZ9D4

1. Consider the following statements:


I. India has emerged as the 5th largest ecosystem for startups globally, with over 70,000 registered
Startups.
II. India is home to 100 unicorns with a total valuation of over USD 300 billion.
Choose the correct option:
a. I only
b. II only

27
c. Both I and II
d. Neither I nor II

2. India and ______ launched the International Solar Alliance on 30th November 2015, on the
side-lines of the UNFCCC. Fill in the blank:
a. UK
b. Australia
c. France
d. USA

3. Consider the following statements regarding the Gulf Cooperation Council:


I. GCC was established by an agreement concluded in 1981 among Bahrain, Kuwait, Oman, Qatar,
Saudi Arabia and UAE.
II. The structure of the GCC consists of the Supreme Council (the highest authority), the
Ministerial Council and the Secretariat General. The Secretariat is located in Abu Dhabi.
Choose the correct option:
a. I only
b. II only
c. Both I and II
d. Neither I nor II

4. The International Parliamentary Union was established in which year?


a. 1924
b. 1896
c. 1889
d. 1947

5. Recently, India and ________ signed a Programme of Cooperation (POC) in the fields of
Science and Technology for the period 2022 – 2025. Fill in the blank:
a. Saudi Arabia
b. Qatar

28
c. UAE
d. Oman

1. B. II only.
2. C. France.
3. A. I only.
4. C. 1889.
5. D. Oman.

'Tomb of Sand' won International Booker Prize

https://unacademy.com/course/mission-35-india-gulf-countries-international-booker/3U5KZ9D4

1. Which book is the first book written in an Indian language to be awarded the International
Booker Prize?

(a) Avatar

(b) Tomb of Sand

(c) Bharat

(d) Nagas

2. Which of the following statements is/are true?

I.VS Naipaul is the first person of Indian Origin to win a Booker Prize.

II.Arundhati Roy is the first Indian Citizen to win the Booker Prize.

(a) I only

(b) II only

(c) Both I and II

(d) Neither I nor II

29
3. Which of the following statements is/are true?

I.The International Booker Prize is awarded annually for a single book, translated into English and
published in Ireland.

II.The International Booker Prize began life in 2005 as the Man Booker International Prize.

(a) I only

(b) II only

(c) Both I and II

(d) Neither I nor II

4. Who is the author of Ret Samadhi?

(a) Geetanjali Shree

(b) Daisy Rockwell

(c) Amish Tripathi

(d) Swaraj Bhatia

5. Who was the first winner of the Man Booker International Prize ?

(a) Jonathan Cape

(b) Kiran Desai

(c) Ismail Kadare

(d) Eleanor Catton

1. (b) Tomb of Sand

2. (c) Both I and II

30
3. (b) II only

4. (a) Geetanjali Shree

5.Answer:(c) Ismail Kadare

Reservation in Promotion

https://unacademy.com/course/mission-35-india-gulf-countries-reservation-in-
promotion/FHGP76EW

1.Who has the right to decide who will be included in the list of Scheduled Caste and Scheduled
Tribes?

(a) President

(b) Prime minister

(c) Parliament

(d) Supreme court

2.Which of the following statements is/are true about the Indra Sawhney Judgment?

I.In the judgment, a nine-judge bench presided by Chief Justice M.H. Kania upheld the
constitutionality of the 27% reservation but put a ceiling of 50% unless exceptional circumstances
warranting the breach

II.The Court directed the exclusion of the creamy layer by way of horizontal division of every
other backward class into creamy layer and non-creamy layer.

(a) I only

(b) II only

(c) Both I and II

31
(d) Neither I nor II

3.Which of the following statements is/are true?

I.The Supreme Court (SC) declared a Maharashtra law which provides reservation benefits to the
Maratha community as constitutional.

II.A 11-member commission headed by Retired Justice N G Gaikwad recommended Marathas


should be given reservation under the Socially and Educationally Backward Class.

(a) I only

(b) II only

(c) Both I and II

(d) Neither I nor II

4.Which of the following is not matched correctly?

(a) Article 338-A-National Commission for Scheduled Tribes

(b) Article 338-National Commission for Scheduled Castes

(c) Article 338-C-National Backward Classes Commission

(d) None of the Above.

5.Which Constitutional amendment Act created the National Commission for SC and ST?

(a) 65th Amendment Act, 1990

(b) 67th Amendment Act, 1991

(c) 69th Amendment Act, 1990

(d) 73rd Amendment Act, 1992

1.Answer:(a) President

32
2.Answer:(c) Both I and II

3.Answer:(b) II only

4.Answer:(c)

5. (a) 65th Amendment Act, 1990

Bilateral Security and Defense Cooperation: Indo-Japan

https://unacademy.com/course/mission-35-indo-japan-relations-un-habitat-plan-for-
jaipur/177G9GDD

1.In which year did the India-Japan Comprehensive Economic Partnership Agreement (CEPA)
come into force?

(a) 2010

(b) 2011

(c) 2012

(d) 2013

2.Which of the following statements is/are true?

I.The Indian PM virtually inaugurated a Japanese ‘Zen Garden - Kaizen Academy’ at Tokyo.

II.In 2020, India and Japan signed a logistics agreement that will allow armed forces of both sides
to coordinate closely in services and supplies.

(a) I only

(b) II only

(c) Both I and II

(d) Neither I nor II

33
3.Which of the following statements is/are true?

I. Supply Chain Resilience Initiative aims to reduce the dependency on a single nation.

II.The initiative, first proposed by India with Japan and Australia as partners, potentially sees other
Asian and Pacific Rim nations later.

(a) I only

(b) II only

(c) Both I and II

(d) Neither I nor II

4.What was the aim of the PM Gati Shakti Scheme?

(a)To ensure integrated planning and implementation of infrastructure projects in the next four
years.

(b) It also aims to have 11 industrial corridors and two new defense corridors.

(c) Both a and b

(d) None of the Above

5.Which of the following bilateral exercises is not held between India and Japan?

(a) JIMEX

(b) SHINYUU Maitri

(c) Dharma Guardian

(d) Shakti Exercise

1.Answer:(b)

2.Answer:(b) II only

34
3.Answer:(a) I only

4.Answer:(c) Both a and b

5.Answer:(d) Shakti Exercise

UN-Habitat Plan for Jaipur

https://unacademy.com/course/mission-35-indo-japan-relations-un-habitat-plan-for-
jaipur/177G9GDD

1. Recently, the UN-Habitat has identified issues like multi hazard vulnerabilities, weak mobility
and Green-Blue economy, associated with which of the following cities?

A. Jaipur

B. Mumbai

C. Indore

D. Trivandrum

2. Which is the first country in the world to launch sovereign Blue Bonds?

A. India

B. Maldives

C. Seychelles

D. Indonesia

3. The United Nations Human Settlements Programme was established in which year?

A. 1973

B. 1975

35
C. 1974

D.1978

4. Consider the following statements”

I. First UN Conference on Human Settlements and Sustainable Urban Development (Habitat I)


held in Vancouver, Canada.

II. First UN Conference on Human Settlements and Sustainable Urban Development (Habitat I)
was held in 1973.

Which of the above statements is correct?

A. Only I

B. Only II

C. Both I & II

D. Neither I nor II

5. The International Labor Organization (ILO) was created in which year?

A. 1919

B. 1920

C. 1913

D. 1921

Answer key

1. A

2. C

3. D

4. A

36
5. A

Cultural Significance of PM’s Gifts for QUAD Leaders

https://unacademy.com/course/mission-35-pms-gift-to-quad-leaders-drone-
mahotsav/NC4754A7

1. The QUAD Summit was held in which city?

A. Tokyo

B. Chennai

C. Seoul

D. Perth

2. Which of the following nations is not a member of the QUAD Grouping?

A. India

B. Japan

C. South Korea

D. Australia

3. The intricate sanjhi panel is based on the theme of Thakurani Ghat, which is one of the most
famous ghats on the banks of which of the following rivers?

A. Ganga

B. Indus

C. Yamuna

D. Narmada

37
4. Consider the following statements regarding Sanjhi Art:

I. Meera, according to Hindu mythology, used to paint sanjhi patterns on the walls for her beloved
Krishna

II. The form was used to make ceremonial rangolis in temples dedicated to Lord Krishna.

Which of the above statements is correct?

A. Only I

B. Only II

C. Both I & II

D. Neither I nor II

5. Deogarh, a classic example of a late Gupta Period type of temple built in the early sixth century
CE, is located in which state?

A. Bihar

B. Jharkhand

C. Rajasthan

D. Uttar Pradesh

Answer Key

1. A

2. C

3. C

4. B

5. D

38
Bharat Drone Mahotsav 2022

https://unacademy.com/course/mission-35-pms-gift-to-quad-leaders-drone-mahotsav/NC4754A7

1. Bharat Drone Mahotsav 2022 was inaugurated in which city?

A. Delhi

B. Hyderabad

C. Bangalore

D. Pune

2. Which of the following schemes launched by the Government of India relates to drone
technology?

A. VAYU scheme

B. UDAAN scheme

C. SVAMITVA scheme

D. GARUD scheme

3. The Ministry notified liberalized drone rules with the aim to encourage R&D and to make India
a drone hub, in which year?

A. 2022

B. 2020

C. 2019

D.2021

4. Startups will be promoted to facilitate which of the following through varied applications and
for Drone-As-A-Service (DrAAS).

39
A. ‘Drone Shakti’

B. 'Atma Nirbhar Bhaarat’

C. ‘Vishwa Guru’

D. None of the above

5. Consider the following statements concerning the PLI Scheme for Drones:

I. The Government allocated Rs. 120 crore for three financial years, in the PLI Scheme.

II. The PLI Scheme for the drones and drone components industry addresses the strategic, tactical,
and operational uses of drone technology.

Which of the above statements is correct?

A. Only I

B. Only II

C. Both I & II

D. Neither I nor II

Answer Key

1. A

2. C

3. D

4. A

5. C

40
India Vietnam Partnership

https://unacademy.com/course/mission-35-india-vietnam-qs-world-university-
rankings/MOJET4FZ

1. Consider the following statements:


I. Recently, the Indian Defense Minister visited Vietnam, where he signed some Defense
Agreements, which will significantly enhance the scope and scale of existing defense cooperation.
II. India and Vietnam are marking 75 years of the establishment of bilateral diplomatic relations.
Choose the correct option:
a. I only
b. II only
c. Both I and II
d. Neither I nor II

2. Which among the following countries is not a part of the Mekong-Ganga Cooperation?
a. Bangladesh
b. Vietnam
c. Cambodia
d. Thailand

3. Which is the capital city of Vietnam?


a. Ho Chi Minh City
b. Hanoi
c. Da Nang
d. Nha Trang

4. Consider the following statements:


I. India Act East Policy was unveiled by Prime Minister of India, Narendra Modi, at the 12th
ASEAN-India Summit in 2014 held in Myanmar. Act East Policy is the successor of Aim East
Policy.

41
II. The Prime minister of India P V Narasimha Rao launched the Aim East policy in 1992, to give
a strategic push to India’s engagement with the South-East Asia region.
Choose the correct option:
a. I only
b. II only
c. Both I and II
d. Neither I nor II

5. Which of the following is not among the 4Cs of Act East Policy?
a. Culture
b. Commerce
c. Connectivity
d. Capability Building

1. Answer: A. I only.
2. Answer: A. Bangladesh.
3. B. Hanoi
4. D. Neither I nor II.
5. D. Capability Building.

QS World University Ranking

https://unacademy.com/course/mission-35-india-vietnam-qs-world-university-
rankings/MOJET4FZ

1. Which among the following is the top university as per QS World University Ranking?
a. Oxford University
b. Stanford University
c. Massachusetts Institute of Technology (MIT)
d. Cambridge University

42
2. The QS World University Ranking is published considering __ parameters. Fill in the blanks:
a. 5
b. 6
c. 8
d. 12

3. Consider the following statements:


I. IIT Bombay was ranked highest among Indian institutes followed by Indian Institute Science
(IISC) and IIT Delhi. The total number of Indian institutes among the top 1,000 globally has risen
to 27 from 22.
II. IISc Bengaluru is the world’s top research university, achieving a perfect score of 100/100 for
this metric.
Choose the correct option:
a. I only
b. II only
c. Both I and II
d. Neither I nor II

4. Which among the following is the fastest rising South Asian University among the QS World
University Rankings top-200?
a. IISC Bengaluru
b. IIT Bombay
c. IIT Madras
d. IIT Delhi

5. Consider the following statements:


I. Quacquarelli Symonds (QS) a leading global career and education network for ambitious
professionals looking to further their personal and professional development.
II. The ‘QS World University Rankings’ is a biennial publication of university rankings which
comprises the global overall and subject rankings.
Choose the correct option:

43
a. I only
b. II only
c. Both I and II
d. Neither I nor II

1. C. Massachusetts Institute of Technology (MIT).


2. B. 6.
3. B. II only.
4. A. IISC.
5. A. I only.

Israel Signs Free Trade Deal with UAE

https://unacademy.com/course/mission-35-israel-uae-trade-deal-thailand-marijuana/EMTSQ20B

1.Which of the following countries is not a part of the West Asian Quad?

(a) Israel

(b) India

(c) UAE

(d) Sudan

2.Which of the following statements is/are true?

I.India-UAE CEPA was signed in 2022, during the India-UAE Virtual Summit.CEPA provides for
an institutional mechanism to encourage and improve trade between the two countries.

II.The India-UAE comprehensive strategic partnership was initiated during the visit of the Prime
Minister of India to UAE in 2019.

(a) I only

44
(b) II only

(c) Both I and II

(d) Neither I nor II

3.Which of the following statements is/are true?

I.Israel signed a free trade deal with the United Arab Emirates, its first with an Arab country,
building on their US-brokered normalization of relations in 2020.

II.The UAE was the second Gulf country to normalize ties with Israel after Jordan.

(a) I only

(b) II only

(c) Both I and II

(d) Neither I nor II

4.Which of the following countries is/are related to the Abraham Accord?

(a) Israel

(b) United Arab Emirates

(c) Bahrain

(d) All of the Above

5.Which of the following countries is/are among the ‘free-trade partners’ of ASEAN?

(a) Japan,

(b) Australia

(c) New Zealand

(d) All of the Above

45
1.Answer:(d) Sudan

2.Answer:(a) I only

3.Answer:(a) I only

4.Answer:(d) All of the Above

5.Answer:(d) All of the Above

Thailand makes Marijuana Legal

https://unacademy.com/course/mission-35-israel-uae-trade-deal-thailand-marijuana/EMTSQ20B

1. Consider the following statements:


I. Recently, Vietnam has legalized cultivating and possessing Marijuana but recreational use (Such
as smoking) is still banned, even though advocates say the easing effectively decriminalizes
Marijuana.
II. Marijuana is a psychoactive drug from the Cannabis plant used for medical, recreational &
religious purposes.
Choose the correct option:
a. I only
b. II only
c. Both I and II
d. Neither I nor II

2. The Narcotic Drugs and Psychoactive Substances (NDPS) Act was enacted in which year?
a. 1980
b. 1985
c. 1990
d. 1995

46
3. Consider the following statements:
I. The Marijuana has two components CBD (cannabidiol) and THC (tetrahydrocannabinol).
II. THC does not cause intoxication or euphoria and has been successfully proven to provide relief
to chronic pain without causing psychoactive side effects. It is CBD which is the primary
psychoactive component of Marijuana.
Choose the correct option:
a. I only
b. II only
c. Both I and II
d. Neither I nor II

4. Who is the current President of the International Narcotics Control Board?


a. Sanjay Kumar Singh
b. Satyesh Bhalla
c. Jagjit Pavadia
d. Satya Narayan Pradhan

5. Consider the following statements:


I. The National Policy on Narcotic Drugs and Psychotropic Substances is based on Article 47 of
the Indian Constitution which directs the State to endeavor to bring about prohibition of the
consumption, except for medicinal purposes, of intoxicating drugs injurious to health.
II. Drug abuse control is the responsibility of state governments.
Choose the correct option:
a. I only
b. II only
c. Both I and II
d. Neither I nor II

1. B. II only.
2. B. 1985.
3. A. I only.

47
4. Answer: C.
5. Answer: A. I only.

India's Trade with the USA

https://unacademy.com/course/mission-35-india-us-trade-tiangong-space-station/OQ3J0LVK

1.Who regulates export/import in India?

(a) Foreign Trade Policy

(b) Export Trade Policy

(c) Directorate General of Trade

(d) None of the Above

2.Which of the following statements is/are true?

I.Bilateral trade between the US and India stood at USD 119.42 billion (2021-2022) as against
USD 80.51 billion in 2020-21.

II.America is one of the few countries with which India has a trade deficit.

(a) I only

(b) II only

(c) Both I and II

(d) Neither I nor II

3.Which of the following statements is/are true?

I.IPEF is a US-led initiative that aims to strengthen economic partnership among participating
countries and competitiveness in the Indo-Pacific region.

48
II.The IPEF was launched with a dozen initial partners who together represent 40% of the world
GDP.

(a) I only

(b) II only

(c) Both I and II

(d) Neither I nor II

4.Which act established the Generalized System of Preferences (GSP)?

(a) Trade Act of 1974

(b) Tariff Act of 1991

(c) Trade Act of 1991

(d) Tariff Act of 1974

5.Which of the following items are exported from India to the US?

(a) Polished diamonds

(b) Gold

(c) Almonds

(d) Rough diamonds

1.Answer:(a) Foreign Trade Policy

2.Answer:(a) I only

3.Answer:(c) Both I and II

4.Answer:(a) Trade Act of 1974

49
5.Answer:(a) Polished diamonds

China’s Tiangong Space Station

https://unacademy.com/course/mission-35-india-us-trade-tiangong-space-station/OQ3J0LVK

1. Consider the following statements:


I. Recently, China’s strategically significant space station project entered the final phase as three
astronauts entered the orbiting module of the Tiangong Space Station.
II. They were launched into the designated orbit by the Zhurong-14 spacecraft.
Choose the correct option:
a. I only
b. II only
c. Both I and II
d. Neither I nor II

2. China launched an unmanned module named ________ for its permanent space station in 2021
that it plans to complete by the end of 2022. Fill in the blanks:
a. Chengdu
b. Tianhe
c. Fengyun
d. Chang’e

3. The China Manned Space Program was launched in which year?


a. 1990
b. 1996
c. 1986
d. 1992

4. Consider the following statements:

50
I. ISS is a habitable artificial satellite - the single largest man-made structure in low earth orbit. Its
first component was launched into orbit in 1998.
II. The ISS programme is a joint project between five participating space agencies: NASA (United
States), Roscosmos (Russia), JAXA (Japan), ESA (Europe), and ASA (Australia) but its ownership
and use has been established by intergovernmental treaties and agreements.
Choose the correct option:
a. I only
b. II only
c. Both I and II
d. Neither I nor II

5. The module, named "Tianhe", or "Harmony of the Heavens", was launched using which rocket?
a. New Glenn
b. Long March 5B
c. Vulcan Centaur
d. Long March 3

1. A. I only.
2. B. Tianhe.
3. D. 1992
4. A. I only.
5. B. Long March 5B

Agneepath Scheme

https://unacademy.com/course/mission-35-agnipath-scheme-sovereign-gold-bond/0HZOFLYD

1. Consider the following statements:


I. Agneepath Scheme allows patriotic and motivated youth to serve in the Armed Forces for a
period of six years.

51
II. Under the new scheme, around 45,000 to 50,000 soldiers will be recruited annually, and most
will leave the service in just four years.
Choose the correct option:
a. I only
b. II only
c. Both I and II
d. Neither I nor II

2. Under this scheme, the youth joining the army will be called:
a. Agni Warrior
b. Agniveer
c. Agnisheel
d. Agni Jawan

3. Under the Agneepath Scheme, aspirants between the ages of 17.5 years and __ years will be
eligible to apply. Fill in the blank:
a. 23
b. 25
c. 27
d. 28

4. Consider the following statements:


I. The 'Agnipath' scheme opens the way for recruitment of about 45,000 soldiers into the Army,
Navy and Air Force in the first year but on a short-term contract of four years. After the completion
of the contract, 25% of them will be retained and the rest will leave the forces.
II. Those hired under the 'Agnipath' scheme will be given a pension benefit when they end their
four-year tenure.
Choose the correct option:
a. I only
b. II only
c. Both I and II

52
d. Neither I nor II

5. Who is the current Chief of Army Staff of the Indian Army?


a. Om Prakash Malhotra
b. Manoj Pandey
c. Bipin Chandra Joshi
d. Manoj Mukund Narvane

1. B. II only.
2. B. Agniveer.
3. C. 21 years.
4. A. I only.
5. B. Manoj Pandey.

Sovereign Gold Bond Scheme

https://unacademy.com/course/mission-35-agnipath-scheme-sovereign-gold-bond/0HZOFLYD

1. Consider the following statement:


I. The Government of India, in consultation with the Reserve Bank of India, will issue Sovereign
Gold Bonds in tranches for 2022-23.
II. The Gold Bonds are issued as Government of India Stock under the Government Securities
(GS) Act, 2006.
Choose the correct option:
a. I only
b. II only
c. Both I and II
d. Neither I nor II

53
2. The SGB scheme was with an objective to reduce the demand for physical gold and shift a part
of the domestic savings - used for the purchase of gold - into financial savings. The scheme was
launched in:
a. 2014
b. 2015
c. 2017
d. 2019

3. Consider the following statements:


I. Gold bond prices are linked to the price of gold of 999 purity (24 carats) published by the India
Bullion and Jewelers Association (IBJA), Mumbai.
II. A fixed rate of 5% per annum is applicable on the scheme, payable semi-annually.
Choose the correct option:
a. I only
b. II only
c. Both I and II
d. Neither I nor II

4. What is the upper limit for retail (individual) investors and HUFs for purchasing gold bonds?
a. 5 Kgs
b. 8 Kgs
c. 10 Kgs
d. 4 Kgs

5. The gold bonds come with a maturity period of:


a. 5 years
b. 3 years
c. 8 years
d. 10 years

1. C. Both I and II.

54
2. B. 2015.
3. A. I only.
4. D. 4 Kgs.
5. C. 8 years.

24th ASEAN India Meeting

https://unacademy.com/course/mission-35-agnipath-scheme-sovereign-gold-bond/0HZOFLYD

1. Consider the following statements:


I. Recently, the 24th ASEAN-India Senior Officials Meeting (SOM) was hosted in Bangalore.
II. The leaders made their assessment on the progress of cooperation under three pillars of
Partnership – Political-Security, Economic and Socio-Cultural.
Choose the correct option:
a. I only
b. II only
c. Both I and II
d. Neither I nor II

2. Which among the following countries is not a member of ASEAN?


a. Brunei
b. Laos
c. India
d. Singapore

3. Consider the following statements:


I. ASEAN is a regional grouping that promotes economic, political, and security cooperation.
II. It was established in August 1976 in Bangkok, Thailand with the signing of the ASEAN
Declaration (Bangkok Declaration).
Choose the correct option:
a. I only

55
b. II only
c. Both I and II
d. Neither I nor II

4. Which among the following countries is not a founding member of ASEAN?


a. Singapore
b. Thailand
c. Malaysia
d. Vietnam

5. How many countries are members of ASEAN?


a. 8
b. 10
c. 12
d. 16

1. B. II only.
2. C. India.
3. A. I only.
4. D. Vietnam.
5. B. 10.

8th International Day of Yoga

https://unacademy.com/course/mission-35-agnipath-scheme-sovereign-gold-bond/0HZOFLYD

1. The 8th International Day of Yoga was celebrated all across the world on:
a. 19th June
b. 21st June
c. 16th June
d. 12th June

56
2. What is the theme of the 8th International Day of Yoga?
a. Yoga for Health
b. Yoga for Humanity
c. Yoga for Better Future
d. Yoga and Humanity

3. Consider the following statements:


I. The idea of International Day of Yoga (IDY) was proposed by India during the opening of the
69th session of the United Nations General Assembly (UNGA), held in 2014.
II. The first Yoga Day celebrations in 2015 at Rajpath in New Delhi.
Choose the correct option:
a. I only
b. II only
c. Both I and II
d. Neither I nor II

4. Consider the following statements:


I. The Prime Minister announced the M-Yoga App which will help in achieving ‘One World One
Health’.
II. The app is a collaboration between the World Health Organization (WHO) and the Ministry of
Health and Family Welfare, Government of India.
Choose the correct option:
a. I only
b. II only
c. Both I and II
d. Neither I nor II

5. Who is the current Minister of Health and Family Welfare?


a. Piyush Goyal
b. Dr. Harsh Vardhan

57
c. Dr. Mansukh Mandaviya
d. Dr. Bharati Pravin Pawar

1. B. 21st June.
2. B. Yoga for Humanity.
3. C. Both I and II.
4. A. I only.
5. C. Dr. Mansukh Mandaviya.

WEB 5.0

https://unacademy.com/course/mission-35-web-50-aspirational-district-program/7ALGYS3G

1. Consider the following statements:


I. Former Twitter CEO Jimmy Butler recently announced his vision for a new decentralized web
platform that is being called Web 5.0
II. It is being built with an aim to return “ownership of data and identity” to individuals.
Choose the correct option:
a. I only
b. II only
c. Both I and II
d. Neither I nor II

2. The web platform is being developed by former Twitter CEO Bitcoin business unit,
__________. Fill in the blank:
a. The Block Head
b. The Block Chain
c. The Block Web
d. The Blockchain Systems

58
3. Consider the following statements:
I. In 1956, American computer scientist John McConaughey organized a conference, at which the
term ‘Artificial Intelligence’ was first adopted.
II. The conference was called the Dartmouth Conference.
Choose the correct option:
a. I only
b. II only
c. Both I and II
d. Neither I nor II

4. Which web was called the “read and write” Internet. Users could now communicate with servers
and other users leading to the creation of the social web. This is the world wide web that we use
today?
a. Web 1.0
b. Web 2.0
c. Web 3.0
d. Web 4.0

1. B. II only.
2. A. The Block Head.
3. B. II only.
4. B. Web 2.0.

Aspirational District Programme

https://unacademy.com/course/mission-35-web-50-aspirational-district-program/7ALGYS3G

1.Which of the following Parameters is not among the Aspirational District Programme key
Ranking Parameters?

(a) Agriculture & Water Resources

59
(b) Health & Nutrition

(c) Financial Inclusion & Skill Development

(d) Women Empowerment

2.Which of the following statements is/are true about the Aspirational District programme?

I. It was launched in 2018 which aims to transform districts that have shown relatively lesser
progress in key social areas.

II.The ADP is essentially aimed at localizing Sustainable Development Goals, leading to the
progress of the nation.

(a) I only

(b) II only

(c) Both I and II

(d) Neither I nor II

3.Which of the following statements is/are true about Delta Ranking?

I.The programme ranks states based on the improvement achieved in a year through the Champions
of Change dashboard (An online Dashboard).

II.States enter data from the previous year after which the dashboard automatically updates ranking
based on the entered data.

(a) I only

(b) II only

(c) Both I and II

(d) Neither I nor II

4.Consider the validity of the following statements about the NITI Aayog.

60
(a) The Chairperson of the Niti Aayog is the Prime Minister.

(b) Chief Ministers of all states and Lt. Governors of Union Territories are part of the Governing
Council.

(c) Both a and b.

(d) Planning Commission was replaced by Niti Aayyog on January 1, 2014.

5.What is 6X6X6 in Anemia Mukt Bharat (AMB) Strategy?

(a) Six target beneficiaries, Six interventions and Six institutional mechanisms for all stakeholders
to implement the strategy.

(b) It is Six doses of Nutrients to Improve Blood.

(c) Both a and b.

(d) None of the above.

1.Answer:(d) Women Empowerment

2.Answer:(c) Both I and II

3.Answer:(d) Neither I nor II

4.Answer:(c) Both a and b.

5.Answer:(a)

India Patent Regime Clash with the U. S. norms

https://unacademy.com/course/mission-35-india-us-patents-issue-jal-jeevan-
mission/XZNIA1WC

1. Consider the following statements:

61
I. Recently, the U.S. Trade Representative highlighted IP challenges in India in its annual Special
201 report.
II. The report highlighted a range of issues in domains ranging from copyright and piracy to
trademark counterfeiting and trade secrets, saying that India remained one of the world’s most
challenging major economies with respect to protection and enforcement of IP.
Choose the correct option:
a. I only
b. II only
c. Both I and II
d. Neither I nor II

2. Which among the following countries is not in the United States’ Priority Watch List?
a. Indonesia
b. Venezuela
c. Argentina
d. Germany

3. Consider the following statements:


I. A copyright is an exclusive set of rights granted for an invention, which may be a product or
process that provides a new way of doing something or offers a new technical solution to a
problem.
II. Indian patents and copyrights are governed by the Indian Patent Act of 1970.
Choose the correct option:
a. I only
b. II only
c. Both I and II
d. Neither I nor II

4. The original Indian Patents Act did not grant patent protection to pharmaceutical products to
ensure that medicines were available to the masses at a low price. This was based on the
recommendations of a 1959 commission chaired by ______. Fill in the blank:

62
a. Sunil Damodar
b. Ajit Tripathi
c. Rajagopala Ayyangar
d. Ramchandra Nair

5. Which Declaration on the TRIPS Agreement and Public Health was adopted by the WTO
Ministerial Conference of 2001 that reaffirmed flexibility of TRIPS member states in
circumventing patent rights for better access to essential medicines?
a. Qatar
b. Doha
c. Istanbul
d. Paris

1. B. II only.
2. D. Germany.
3. D. Neither I nor II.
4. D.
5. B. Doha Declaration.

Jal Jeevan Mission

https://unacademy.com/course/mission-35-india-us-patents-issue-jal-jeevan-
mission/XZNIA1WC

Q. 1 Jal Jeevan Mission envisages the supply of _____ liters of water per person per day to every
rural household through Functional Household Tap Connections (FHTC) by 2024.
(a) 50
(b) 55
(c) 60
(d) 65

63
Q. 2 At the state level, which of the following states has not achieved 100% tap connectivity to all
households in the state?
(a) Goa
(b) Telangana
(c) Haryana
(d) Tamil Nadu

Q. 3 Jal Jeevan Mission (Urban) has been announced by the Housing and Urban Affairs Ministry
under which of the following sustainable goals?
(a) Sustainable Goal 3
(b) Sustainable Goal 4
(c) Sustainable Goal 5
(d) Sustainable Goal 6

Q. 4 Consider the following statements in relation to the significance of Sustainable Developments


2030.
I. Sustainable development means development which meets the needs of the present without
compromising the ability of future generations to meet their own needs.
II. The most widely accepted definition of Sustainable Development was given by the Brundtland
Commission in its report Our Common Future (1987).
Which of the above statements is/are incorrect?
(a). Only I
(b) Only II
(c) Both I and II
(d) Neither I and II

Q. 5 Consider the following statements in relation to the significance of the Union Budget under
the Indian Constitution.
I. Budget is a statement of the estimated receipts and expenditure of the Government in a Financial
Year (which begins on 1st April of the current year and ends on 31st March of the following year).

64
II. According to Article 102 of the Indian Constitution, the Union Budget of a year is referred to
as the Annual Financial Statement (AFS).
Which of the above statements is/are incorrect?
(a) Only I
(b) Only II
(c) Both I and II
(d) Neither I and II

Answers:
1. (b) 55
2. (d) Tamil Nadu
3. (d) Sustainable Goal 6
4. (d) d. Neither I and II
5. (b) Only II

14th BRICS Summit

https://unacademy.com/course/mission-35-brics-summit-one-candidate-one-
constituency/W97WVO8D

1. Recently, the Prime Minister of India attended the 14th BRICS summit which was virtually
hosted by:
a. Brazil
b. Russia
c. China
d. South Africa

2. Consider the following statements:


I. Theme of the 14th BRICS Summit: Ushering a New Era of High-quality BRICS Partnership.

65
II. The BRICS Plus virtual conference was also held as part of the main meeting with ministers
from countries, including the UAE, Saudi Arabia, Egypt, Kazakhstan, Indonesia, Argentina,
Nigeria, Senegal, and Thailand.
Choose the correct option:
a. I only
b. II only
c. Both I and II
d. Neither I nor II

3. In 2001, the British Economist ________ coined the term BRIC to describe the four emerging
economies of Brazil, Russia, India, and China. Choose the correct option:
a. Jim O’ Neill
b. J. A. Hobson
c. Charles Babbage
d. Philip Andrews

4. Consider the following statements:


I. During the Sixth BRICS Summit in Fortaleza (Brazil) in 2014, the leaders signed the Agreement
establishing the New Development Bank in Beijing, China.
II. The chairmanship of the BRICS forum rotates annually among the members, in accordance
with the acronym B-R-I-C-S.
Choose the correct option:
a. I only
b. II only
c. Both I and II
d. Neither I nor II

5. South Africa became a member of BRIC in which year?


a. 2006
b. 2008
c. 2010

66
d. 2012

1. C. China
2. B. II only.
3. A. Jim O’ Neill
4. B. II only.
5. C. 2010

One Candidate One Constituency

https://unacademy.com/course/mission-35-brics-summit-one-candidate-one-
constituency/W97WVO8D

1. Who is the current Chief Election Commissioner of India?


a. Sunil Chandra
b. Rajiv Kumar
c. Navin Chawla
d. Sushil Tandon

2. Consider the following statements:


I. An opinion poll is a pre-election survey to gather voters’ views on a range of election-related
issues.
II. An exit poll, on the other hand, is conducted immediately before people have voted, and assesses
the support for political parties and their candidates.
Choose the correct option:
a. I only
b. II only
c. Both I and II
d. Neither I nor II

67
3. Which Section of the Representation of People’s Act, provides the number of constituencies
from which a candidate can contest elections?
a. Section 42(2)
b. Section 54
c. Section 33(7)
d. Section 23(4)

4. The RPA was amended in which year to set a cap at a maximum of two constituencies.
a. 1994
b. 1996
c. 1976
d. 1972

5. Which Article of the Indian Constitution provides for the establishment of the Election
Commission of India?
a. Article 324
b. Article 234
c. Article 124
d. Article 224

1. B. Rajiv Kumar
2. A. I only.
3. C. Section 33(7).
4. B. 1996.
5. A. Article 324.

Governor’s Power to Call for a Floor Test

https://unacademy.com/course/mission-35-floor-test-stockholm50/0WS714V3

68
1. Which Article of the Constitution authorizes the Governor to summon, dissolve and prorogue
the state legislative assembly?
a. Article 174
b. Article 170
c. Article 168
d. Article 162
Answer: A

2. Consider the following statements:


I. When the House is in session, it is the Speaker who can call for a floor test. But when the
Assembly is not in session, the Governor’s residuary powers under Article 163 allow him to call
for a floor test.
II. The Governor cannot exercise his discretionary power under Article 174, when the chief
minister has lost the support of the House and his strength is debatable.
Choose the correct option:
a. I only
b. II only
c. Both I and II
d. Neither I nor II
Answer: A

3. Who is the current Governor of Maharashtra?


a. Shankar Dayal Sharma
b. S. M. Krishna
c. Bhagat Singh Koshyari
d. Mohammed Fazal
Answer: C

4. Consider the following statements:


I. Floor Test is a term used for the test of the majority. If there are doubts against the Chief Minister
(CM) of a State, he/she can be asked to prove the majority in the House.

69
II. In the absence of a clear majority, when there is more than one individual staking claim to form
the government, the Governor may call for a special session to see who has the majority to form
the government.
Choose the correct option:
a. I only
b. II only
c. Both I and II
d. Neither I nor II
Answer: C

5. Which Article of the Constitution deals with the Rules of Procedure in State Legislatures?
a. Article 175
b. Article 163
c. Article 176
d. Article 208
Answer: D

1. A. Article 174.
2. A. I only.
3. C. Bhagat Singh Koshyari.
4. C. Both I and II.
5. D. Article 208.

Stockholm+50

https://unacademy.com/course/mission-35-floor-test-stockholm50/0WS714V3

1.Which country first proposed the idea of the Stockholm Conference?

(a) Australia

(b) New Zealand

70
(c) Japan

(d) Sweden

2.Which of the following statements is/are true?

I. The UNFCCC, signed in 1992 at the United Nations Conference on Environment and
Development also known as the Earth Summit, the Rio Summit or the Rio Conference.

II.The COP is the “supreme body” of the Convention, as it is its highest decision-making authority.
The climate change process revolves around the annual sessions of the COP.

(a) I only

(b) II only

(c) Both I and II

(d) Neither I nor II

3.Which of the following statements is/are true about the Paris Climate Accord?

I.It was adopted by 196 countries at Conference of the Parties COP 21 in Paris in December 2014.

II.Its Goal is to limit global warming to well below 2° Celsius, and preferably limit it to 1.5°
Celsius, compared to pre-industrial levels.

(a) I only

(b) II only

(c) Both I and II

(d) Neither I nor II

4.In which year India set up its ministry of environment and forest?

(a) 1950

(b) 1985

71
(c) 1965

(d) 1946

5.Which of the following reports is not published by the United Nations Environment Programme?

(a) Emission Gap Report

(b) Adaptation Gap Report

(c) Global Environment Outlook

(d) Global Hunger Index

1.Answer:(d) Sweden

2.Answer:(c) Both I and II

3.Answer:(b) II only

4.Answer:(b) 1985

5.Answer:(d) Global Hunger Index

Goods and Services Tax Council

https://unacademy.com/course/mission-35-stockholm-50-gst-council/3E8AKROL.

1. Consider the following statements:


I. The GST Council is a central body meant to “make recommendations to the Union and the states
on important issues related to GST, like the goods and services that may be subjected or exempted
from GST, model GST Laws”.
II. The Goods and Services Tax has subsumed indirect taxes like excise duty, Value Added Tax
(VAT), service tax, luxury tax etc.
Choose the correct option:
a. I only

72
b. II only
c. Both I and II
d. Neither I nor II

2. The GST Council was set up as per _________ of the Constitution. Fill in the blanks:
a. Article 279
b. Article 263A
c. Article 273A
d. Article 279A

3. Consider the following statements:


I. The members of the Council include the Union Finance Minister, the Union Minister of State
(Finance) from the Center.
II. The Union Finance Secretary is the Chairperson of the GST Council.
Choose the correct option:
a. I only
b. II only
c. Both I and II
d. Neither I nor II

4. Consider the following statements:


I. As per the Supreme Court, the recommendations of the GST Council are binding.
II. GST has a 2-tier tax structure for all goods and services under the slabs- 18% and 28%.
Choose the correct option:
a. I only
b. II only
c. Both I and II
d. Neither I nor II

5. GST was introduced through the 101st Constitution Amendment Act in:
a. 2014

73
b. 2015
c. 2016
d. 2018

1. B. II only.
2. D. Article 279A
3. A. I only
4. D. Neither I nor II
5. C. 2016

48th G7 Summit

https://unacademy.com/course/mission-35-g7-summit-eco-sensitive-zone/TN3BNQNE.

1. Which country holds the presidency of G7 in 2022?


a. India
b. China
c. Germany
d. Canada

2. Which among the following countries is not a part of G7?


a. Canada
b. Italy
c. Japan
d. Australia

3. Consider the following statements:


I. G7 announced the collective mobilization of 600 billion dollars by 2027 under Partnership for
Global Infrastructure and Investment (PGII) to deliver “game-changing” and “transparent”
infrastructure projects to developing and middle-income countries.
II. G7 secretariat is situated in New York.

74
Choose the correct option:
a. I only
b. II only
c. Both I and II
d. Neither I nor II

4. Consider the following statements:


I. Recently, the Ministry of Power (MoP) announced a Green Hydrogen Policy (GHP). It is a
watershed moment in India’s energy transition journey, and by doing so, India has become the
18th country to release a comprehensive Green Hydrogen Policy.
II. India has committed to achieving net-zero carbon emissions by 2050.
Choose the correct option:
a. I only
b. II only
c. Both I and II
d. Neither I nor II

5. Who is the current Minister of Power and Energy?


a. Nitin Jairam Gadkari
b. Piyush Goyal
c. Raj Kumar Singh
d. Ashish Upadhyaya

1. C. Germany
2. D. Australia
3. A. I only.
4. A. I only.
5. C. Raj Kumar Singh

75
SC Judgment on Eco Sensitive Zone

https://unacademy.com/course/mission-35-g7-summit-eco-sensitive-zone/TN3BNQNE

1.Which of the following activities is not permitted in Eco Sensitive Zones?

(a) Ongoing agricultural

(b) Rainwater harvesting

(c) Organic farming

(d) Establishment of major hydroelectric projects

2.Which of the following statements is/are true?

I. The Supreme Court directed that every protected forest, national park and wildlife sanctuary
across the country should have a mandatory eco-sensitive zone (ESZ) of a minimum two km
starting from their demarcated boundaries.

II.The Center had, while coming out with 2011 guidelines on ESZ, had prescribed a 10-kilometer
boundary based on responses received from states and UTs.

(a) I only

(b) II only

(c) Both I and II

(d) Neither I nor II

3.Which of the following statements is/are true about Kaziranga National Park?

I.It was declared as a National Park in 1974.It has been declared a tiger reserve since 2007.

II.It was declared a UNESCO World Heritage Site in 1985.

(a) I only

(b) II only

76
(c) Both I and II

(d) Neither I nor II

4.Which Conference inspired India to enact the Environment (Protection) Act, 1986?

(a) Stockholm Conference

(b) COP 18

(c) BIMSTEC

(d) Asia-Pacific Economic Cooperation

5.Which of the following Offense/Offenses is punishable under Wildlife Protection Act, 1972?

(a) Altering boundaries of a Sanctuary or National Park.

(b) Hunting of wild animals.

(c) Harvesting the wild animals.

(d) All of the Above.

1.Answer:(d) Establishment of major hydroelectric projects

2.Answer:(b) II only

3.Answer:(c) Both I and II

4.Answer:(a) Stockholm Conference

5.Answer:(d) All of the Above.

Transfer of in-orbit Communication to NewSpace India Ltd (NSIL)

https://unacademy.com/course/mission-35-eco-sensitive-zone-nsil/KP3TX185

1. Recently, the government has approved the transfer of how many in-orbit communication
satellites from the Government of India to NewSpace India Ltd (NSIL)?

77
A. 10

B. 14

C. 17

D. 15

2. The NSIL was established in which year?

A. 2012

B. 2019

C. 2017

D. 2016

3. Consider the following statements regarding the ATL Space Challenge.

I. It was launched in 2019

II. This mission has been launched by NITI Aayog’s Atal Innovation Mission (AIM) in
collaboration with ISRO (Indian Space Research Organisation) and ICSE.

Which of the above statements is correct?

A. Only I

B. Only II

C. Both I & II

D. Neither I nor II

4. Consider the following statements regarding NSIL.

I. It is headquartered at Bangalore

II. It is the commercial arm of Indian Space Research Organization (ISRO)

78
Which of the above statements is correct?

A. Only I

B. Only II

C. Both I & II

D. Neither I nor II.

5. The NAVIC system was launched by the Indian Space Research Organisation (ISRO) with
which of the following?

A. IIT Bombay

B. NewSpace India Ltd.

C. Antrix Corporation Ltd.

D. None of the above

Answer Key

1. A

2. B

3. D

4. C

5. C

UN Report on the Taliban Regime

https://unacademy.com/course/mission-35-un-taliban-report-india-tajikistan/GH5AKU5W

79
1.Which of the following statements is/are true about the United Nations Security Council?

(a) The council has ten permanent members.

(b) India, for the eighth time, entered the UNSC as a non-permanent member last year (2021).

(c) Each year, the General Assembly elects five permanent members (out of ten in total) for a two-
year term.

(d) Both a and c.

2.Which of the following statements is/are true about the United Nations Security Council
(UNSC)’s Analytical Support and Sanctions Monitoring Team ?

I. India is currently the chair of the UNSC sanctions committee, which comprises all the 15 UNSC
members.

II.UNAMA is a UN Special Political Mission established to assist the state and the people of
Afghanistan in laying the foundations for sustainable peace and development.

(a) I only

(b) II only

(c) Both I and II

(d) Neither I nor II

3.Which of the following statements is/are true?

I. The 21st Summit of the Shanghai Cooperation Organisation (SCO) Council of Heads of State
was held recently via video Conferencing in Dushanbe, Tajikistan.At the meeting the focus was
primarily on the ground situation in Afghanistan and its global repercussions.

II.Shanghai Cooperation Organization is a permanent intergovernmental international


organisation.It was created in 2007.

(a) I only

80
(b) II only

(c) Both I and II

(d) Neither I nor II

4.Which of the following dams is/are built by India in Afghanistan?

(a) Shahtoot Dam

(b) Salma Dam

(c) Both a and b

(d) Dahla Dam.

5.Troika Plus Meeting on Afghanistan includes-

(a) U.S

(b) Russia

(c) China

(d) All of the Above

1.Answer:(b)

2.Answer:(c) Both I and II

3.Answer:(a) I only

4.Answer:(c) Both a and b

5.Answer:(d) All of the Above

India Tajikistan Bilateral Relations

https://unacademy.com/course/mission-35-un-taliban-report-india-tajikistan/GH5AKU5W

81
1. Which among the following countries does not share a land border with Tajikistan?
a. Afghanistan
b. Turkmenistan
c. China
d. Uzbekistan

2. Consider the following statements:


I. Kazakhstan, Kyrgyzstan, Tajikistan, Turkmenistan and Uzbekistan became independent states
after the collapse of the USSR post-Cold war.
II. All these countries are also members of the Shanghai Cooperation Organization.
Choose the correct option:
a. I only
b. II only
c. Both I and II
d. Neither I nor II

3. Which among the following is the capital city of Tajikistan?


a. Tashkent
b. Dushanbe
c. Ashgabat
d. Vahdat

4. Consider the following statements:


I. Recently, Raj Kumar Singh, India’s Minister for Science and Technology and Earth Sciences
held a bilateral meeting with the Minister for Energy and Water resources of Republic of
Tajikistan.
II. Issues like water resources research especially Glacier monitoring, Non-conventional Energy,
Peaceful Use of Space Technology and Disaster Management were discussed to support global
water action and climate resistance on Water for Sustainable Development.
Choose the correct option:

82
a. I only
b. II only
c. Both I and II
d. Neither I nor II

5. Tajikistan became independent in which year?


a. 1950
b. 1948
c. 1979
d. 1991

1. B. Turkmenistan
2. A. I only
3. B. Dushanbe
4. B. II only
5. D. 1991

SHRESHTA Scheme

https://unacademy.com/course/mission-35-shrestha-scheme-horn-of-africa/YBZ7JVNJ

1.In which year Post Matric scholarship schemes for SC were launched?

(a) 1996

(b) 2001

(c) 2006

(d) 2011

2.Which of the following statements is/are true?

83
I. The Ministry of Social Justice and Empowerment has launched the scheme ‘SHRESHTA.’ This
scheme is known as the Scheme for residential education for students in High school in Targeted
Areas.

II.Its basic motive is to uplift the socio-economic status of the Scheduled Castes and Scheduled
tribes peoples by providing high quality education to their children in best private government
schools in the country.

(a) I only

(b) II only

(c) Both I and II

(d) Neither I nor II

3.Which of the following statements is/are true about the National Testing Agency?

I.National Testing Agency (NTA) was established as a Society registered under the Indian
Societies Registration Act, 1860.

II.NTA is chaired by an eminent educationist appointed by the Ministry of Human Resource


Development.

(a) I only

(b) II only

(c) Both I and II

(d) Neither I nor II

4.In which class Admission will be provided under the ‘SHRESHTA’ Scheme?

(a) Class 6

(b) Class 9

(c) Class 11

84
(d) Both b and c

5.When was the first Socio-Economic and Caste Census conducted?

(a) 1860

(b) 1945

(c) 1918

(d) 1931

1.Answer:(c) 2006

2.Answer:(a) I only

3.Answer:(c) Both I and II

4.Answer:(d) Both b and c

5.Answer:(d) 1931

China’s Presence in the Horn of Africa 0

https://unacademy.com/course/mission-35-shrestha-scheme-horn-of-africa/YBZ7JVNJ

1. Consider the following statements:


I. The Horn of Africa is a peninsula in Northeast Africa. Located on the westernmost part of the
African mainland, it is the fourth largest peninsula in the world.
II. The area has experienced imperialism, neo-colonialism, Cold War, ethnic strife, intra-African
conflict, poverty, disease, famine and much more.
Choose the correct option:
a. I only
b. II only
c. Both I and II

85
d. Neither I nor II

2. Which among the following countries do not make up the Horn of Africa?
a. Djibouti
b. Ethiopia
c. Kenya
d. Somalia

3. Consider the following statements:


I. In January 2022, China asserted its three objectives in Africa: controlling the pandemic,
implementing a Forum on Cooperation between Africa and China (FOCAC) outcomes, and
upholding common interests while fighting hegemonic politics.
II. China-Africa Cooperation Vision 2035 was formulated to determine the directions and
objectives of mid- and long-term cooperation and promote a closer community with a shared future
for China and Africa.
Choose the correct option:
a. I only
b. II only
c. Both I and II
d. Neither I nor II

4. China’s first and only military base outside its mainland is in:
a. Sudan
b. Ethiopia
c. Djibouti
d. Rwanda

5. Which among the following is the largest country in Africa?


a. Sudan
b. Algeria
c. Democratic Republic of Congo

86
d. Libya

1. B. II only
2. C. Kenya
3. C
4. C. Djibouti
5. B. Algeria

India's First Liquid Mirror Telescope

https://unacademy.com/course/mission-35-wto-liquid-mirror-telescope/41PAWQVB

1.Which of the following countries is not involved in International Liquid-Mirror Telescope


development?

(a) India

(b) Canada

(c) Poland

(d) Russia

2.Which of the following statements is/are true?

I.The Devasthal Observatory campus owned by Aryabhatta Research Institute of Observational


Sciences (ARIES), Nainital in Uttarakhand has set-up the International Liquid-Mirror Telescope.

II.International Liquid-Mirror Telescope. has become the world's first Liquid-Mirror Telescope
(LMT) to be commissioned for astronomy and also one of its kind to be operational anywhere in
the world.

(a) I only

(b) II only

87
(c) Both I and II

(d) Neither I nor II

3.Which of the following statements is/are true?

I. Devasthal is one of the world’s original sites for obtaining astronomical observations.

II.ILMT will be the third telescope facility to come up at Devasthal.

(a) I only

(b) II only

(c) Both I and II

(d) Neither I nor II

4.What is the purpose of EOS-01?

(a) Agriculture

(b) Forestry

(c) Disaster management support

(d) All of the Above.

5. Which is India's first experimental satellite launch vehicle?

(a) GSLV

(b) ASLV

(c) SLV 3

(d) None of these

1.Answer:(d) Russia

2.Answer:(c) Both I and II

88
3.Answer:(c) Both I and II

4.Answer:(d) All of the Above.

5.Answer:(c) SLV 3

New POEM Platform

https://unacademy.com/course/mission-35-poem-platform-inter-state-meet/2DNBVL0H

1. Consider the following statements:


I. Recently, the Indian Space Research Organization (ISRO) achieved the feat of successfully
launching the PSLV Orbital Experimental Module or ‘POEM’.
II. ISRO also launched three satellites from Satish Dhawan Space Center on the PSLV-C53.
Choose the correct option:
a. I only
b. II only
c. Both I and II
d. Neither I nor II

2. NewSpace India Limited was established in which year?


a. 2014
b. 2016
c. 2017
d. 2019

3. Consider the following statements:

89
I. POEM (PSLV Orbital Experimental Module) is an experimental mission by ISRO which
performs in-orbit scientific experiments during the third stage of the Polar Satellite Launch Vehicle
(PSLV) launch vehicle as an orbital platform.
II. The PSLV is a three-stage rocket where the first two spent stages fall back into the ocean, and
the final stage — after launching the satellite into orbit — ends up as space junk.
Choose the correct option:
a. I only
b. II only
c. Both I and II
d. Neither I nor II

4. Who is the current Chairman of ISRO?


a. K Sivan
b. S. Somnath
c. A. S. Kiran Kumar
d. U. R. Rao

5. Which among the following is the first launch vehicle to be equipped with liquid stages?
a. PSLV
b. GSLV
c. ASLV
d. SSLV

1. A. I only
2. D. 2019
3. D. Neither I nor II
4. B. S Somnath
5. A. PSLV

90
Ban on Export of Sugar

https://unacademy.com/course/mission-35-export-ban-on-sugar-un-oceans-
conference/CWQJ7GUA

1. Consider the following statements regarding Agriculture Infrastructure Development Cess:

I. The purpose of the new AIDC is to raise funds to finance spending on developing agriculture
infrastructure.

II. Cess is a kind of special-purpose tax which is levied over and above basic tax rates.

Which of the above statements is correct?

A. Only I

B. Only II

C. Both I & II

D. Neither I nor II

2. The restriction on sugar is not applicable for exports to which of the following:

A. European Union

B. United States

C. African Continent

D. Both A & B

3. Russia annexed Crimea in which year?

A. 2014

B. 2015

C. 2016

D. 2017

91
4. North Atlantic Treaty Organization (NATO) is a military alliance established by the North
Atlantic Treaty (also called the Washington Treaty, enacted in which year?

A. 1942

B. 1945

C. 1947

D. 1949

5. Which country is the world’s biggest producer, exporter, and consumer of palm oil?

A. India

B. China

C. Bahamas

D. Indonesia

Answer Key

1. C

2. D

3. A

4. D

5. D

National Investigation Agency

92
https://unacademy.com/course/mission-35-nia-world-health-assembly/YWAHS95S

1. The National Investigation Agency has been established under which of the following Acts?
a. National Investigation Agency, 2007
b. National Investigation Agency, 2008
c. National Investigation Agency, 2009
d. National Investigation Agency, 2010

2. The NIA Special Court shall be presided over by a judge to be appointed by the Central
Government on the recommendation of the ________.
a. Chief Justice of India
b. President of India
c. Chief Justice of the High Court
d. Governor of the concerned state

3. Under which Section of the NIA Act, 2008, State governments can refer the cases pertaining to
the scheduled offenses registered at any police station to the Central government for NIA
investigation?
a. Section 4
b. Section 5
c. Section 6
d. Section 7

4. Consider the following statements in relation to the historical significance of the United Nations
(UN).
I. The United Nations (UN) is an international organization founded in 1945 and is currently made
up of 193 Member States.
II. The forerunner of the United Nations was the League of Nations, an organization conceived in
the circumstances of the First World War, and established in 1919 under the Treaty of Versailles.

Which of the following statements are incorrect?

93
a. Only I
b. Only II
c. Both I and II
d. Neither I and II

5. Consider the following statements in relation to the much contested Unlawful Activities
Prevention Act, 1967.
I. Under the UAPA, the investigating agency can file a charge sheet in maximum 90 days after the
arrests and the duration can be extended further after intimating the court.
II. Under UAPA, both Indian and foreign nationals can be charged.
Which of the following statements are incorrect?
a. Only I
b. Only II
c. Both I and II
d. Neither I and II

Answers:
1. B
2. C
3. C
4. D
5. A

75th Session of World Health Assembly

https://unacademy.com/course/mission-35-nia-world-health-assembly/YWAHS95S

1. The headquarters of the World Health Organization (WHO) is located at which of the following
places?

A. Geneva

94
B. The Hague

C. Washington D.C.

D. Paris

2. What was the theme of the World Health Assembly, 2022?

A. Health and Wellness – Mental welfare

B. Striving for Health

C. Health for peace, peace for health

D. None of the above

3. The National Health Mission (NHM) was launched by the government of India in which year?

A. 2013

B. 2012

C. 2015

D. 2014

4. Consider the following statements regarding the functions of the World Health Assembly:

I. Appointment of the Director-General of WHO.

II. Review and approval of the proposed programme budget.

Which of the above statements is correct?

A. Only I

B. Only II

C. Both I & II

D. Neither I nor II.

95
5. The Registrar General of India was founded in which year?

A. 1950

B. 1961

C. 1964

D. 1965

Answer Key

1. A

2. C

3. A

4. C

5. B

Coalition for Disaster Resilient Infrastructure (CDRI)

https://unacademy.com/course/mission-35-world-health-assembly-cdri/M9JDXOZG
1. How many countries are currently members of the Coalition for Disaster Resilient Infrastructure
(CDRI) which has recently been recognized as an international organization by the Union
Government?
A. 30
B. 31
C. 32
D. 35

2. Coalition for Disaster Resilient Infrastructure (CDRI) was launched in ________at the United
Nations Climate Action Summit in New York.

96
A. 2018
B. 2019
C. 2020
D. 2021

3. Consider the following statements in relation to the significance of the Asian Development Bank
(ADB).
I. ADB is a regional development bank established in 1966 with India as a founding member.
II. It is headquartered in Manila, Philippines.

Which of the above statements is/are incorrect?


A. Only I
B. Only II
C. Both I and II
D. Neither I and II

4. Consider the following statements in relation to India's Global Initiative of International Solar
Alliance (ISA).
I. The ISA was conceived as a joint effort by India and Germany to mobilize efforts against climate
change through deployment of solar energy solutions.
II. The Headquarters is in India with its Interim Secretariat being set up in Gurugram.

Which of the above statements is/are incorrect?


A. Only I
B. Only II
C. Both I and II
D. Neither I and II

5. Which of the following Acts provide that Coalition for Disaster Resilient Infrastructure (CDRI)
member's property and assets wherever located and by whomsoever held, shall enjoy immunity

97
from every form of legal process except in any particular case it has expressly waived its
immunity?
A. United Nations (Privileges and Immunities) Act, 1946
B. United Nations (Privileges and Immunities) Act, 1947
C. United Nations (Privileges and Immunities) Act, 1948
D. United Nations (Privileges and Immunities) Act, 1949

Answers
1. B
2. B
3. D
4. A
5. B

Australia India Water Security Initiative

https://unacademy.com/course/mission-35-indo-aus-water-initiative-vp-of-india/EV90F975

1. Consider the following statements:


I. AIWASI is a project under the South East Asia Water Security Initiative (SEAWASI) of the
Department of Foreign Affairs and Trade (DFAT), Australia.
II. It aims to work towards the Water Sensitive City vision which is based on holistic management
of the integrated water cycle.
Choose the correct option:
a. I only
b. II only
c. Both I and II
d. Neither I nor II

2. Who is the current Minister of Jal Shakti?


a. Narayan Rane

98
b. Nitin Gadkari
c. Gajendra Singh Shekhawat
d. Narendra Singh Tomar

3. The Jal Shakti Abhiyaan was launched in which year?


a. 2014
b. 2016
c. 2017
d. 2019

4. Consider the following statements regarding Composite Water Management Index:


I. NITI Aayog first launched and conceptualized the Composite Water Management Index in 2018
as a tool to instill a sense of cooperative and competitive federalism among the states. The index
comprises 12 themes.
II. CWMI 2.0 ranks various states for the reference year 2017-18 as opposed to the base year 2016-
17.
Choose the correct option:
a. I only
b. II only
c. Both I and II
d. Neither I nor II

5. Which state continues to hold on to its rank one since the reference year?
a. Gujarat
b. Andhra Pradesh
c. Kerala
d. Goa

1. B. II only.

99
2. C. Gajendra Singh Shekhawat.
3. D. 2019.
4. B. II only.
5. A. Gujarat.

Election to the Office of Vice President in India

https://unacademy.com/course/mission-35-indo-aus-water-initiative-vp-of-india/EV90F975

1. Under which Article of the Indian Constitution, the Vice-President is elected by the members
of the Electoral College?
A. Article 60
B. Article 62
C. Article 64
D. Article 66

2. Which of the following members are not a part of the electoral college for the election of Vice
President in India?
A. Elected members of Rajya Sabha
B. Nominated members of Rajya Sabha
C. Nominated members of Lok Sabha
D. Elected members of Lok Sabha

3. Article _______of the Constitution read with the Presidential and Vice-Presidential Elections
Act, 1952 and the Presidential and Vice-Presidential Elections Rules, 1974, vests the
superintendence, direction and control of the conduct of election to the office of the Vice-President
of India in the Election Commission of India.
A. Article 324
B. Article 374
C. Article 234
D. Article 254

100
4. Consider the following statements in relation to the significance of the Election Commission in
India.
I. The Election Commission was established in accordance with the Constitution on 25th January
1950.
II. Originally the commission had only one election commissioner but after the Election
Commissioner Amendment Act 1985, it has been made a multi-member body.
Which of the above statements is/are incorrect?
A. Only I
B. Only II
C. Both I and II
D. Neither I and II

5. Consider the following statements in relation to the historical significance of the Rajya Sabha
in India.
I. The Upper House of the Parliament, Rajya Sabha or Council of States was constituted on 3rd
April 1952 and the first session was held on 13th May 1952.
II. The origin of the Rajya Sabha or the Second Chamber can be traced to the Montague-
Chelmsford Report of 1918.
Which of the above statements is/are correct?
A. Only I
B. Only II
C. Both I and II
D. Neither I and II

Answer:
1. D
2. C
3. A
4. B
5. C

101
China’s new High-Tech Aircraft Carrier Fujian

https://unacademy.com/course/mission-35-fujian-indian-railways-innovation/6KTFAIIH

1. Recently, which of the following countries unveiled its first indigenous aircraft carrier, the new-
generation Fujian (Type 003)?
A. Japan
B. China
C. South Korea
D. North Korea

2. Consider the following statements in relation to the historical significance of the INS
Vikramaditya.
I. INS Vikramaditya is the Indian Navy’s largest aircraft carrier and warship converted from the
Russian Navy’s decommissioned Admiral Gorshkov/Baku.
II. It is a modified Kiev-class aircraft carrier that was commissioned into service in November
2018.
Which of the above statements is/are correct?
A. Only I
B. Only II
C. Both I and II
D. Neither I and II

3. Which of the following countries has firstly developed the Electromagnetic Aircraft Launch
System (EMALS) which has been fitted in China's new Aircraft Carrier Fujian?
A. USA
B. Japan
C. Russia
D. Australia

102
4. Consider the following statements in relation to the significance of Trincomalee Port in Sri
Lanka.
I. Trincomalee harbor is on the northeastern coast of Sri Lanka and is situated on a peninsula in
Trincomalee Bay—formerly called Koddiyar Bay.
II. China commissioned the 2020 ADB (Asian Development Bank) study on the development of
Trincomalee port.
Which of the above statements is/are correct?
A. Only I
B. Only II
C. Both I and II
D. Neither I and II

5. India’s second aircraft carrier named INS Vikrant, set to be commissioned later this year, will
use which of the following systems to launch aircraft?
A. Short Take-Off But Arrested Recovery (STOBAR)
B. Catapult Assisted Take-off But Arrested Recovery (CATOBAR)
C. Both A and B
D. None of the above

Answers:
1. B
2. A
3. A
4. A
5. B

Indian Railways Innovation Policy

https://unacademy.com/course/mission-35-fujian-indian-railways-innovation/6KTFAIIH

1. Consider the following statements:

103
I. Recently, the Minister of Railways, Piyush Goyal has launched the Indian Railway Innovation
Policy- “StartUps for Railways”.
II. The entire process, starting from the floating of the problem statement to development of the
prototype, will be online with a defined time line to make it transparent and objective.
Choose the correct option:
a. I only
b. II only
c. Both I and II
d. Neither I nor II

2. Consider the following statements:


I. Under the policy, grant up to Rs. 1.5 Crore to innovators on an equal sharing basis with provision
of milestone-wise payment.
II. The developed Intellectual Property Rights will be transferred to the Railway Ministry.
Choose the correct option:
a. I only
b. II only
c. Both I and II
d. Neither I nor II
3. The Indian Railway was introduced in ____, when a line was constructed from Bombay to Thane
covering a distance of 34 km. Fill in the blank:
a. 1853
b. 1854
c. 1850
d. 1857

4. The Indian Railway System has been divided into how many zones?
a. 12
b. 24
c. 18
d. 16

104
5. Which among the following is not among the UNESCO accorded World Heritage Sites?
a. Matheran Light Railway
b. Chhatrapati Shivaji Terminus, Mumbai
c. Darjeeling Himalayan Railway
d. Kalka Shimla Railway

1. B. II only.
2. A. I only.
3. A. 1853.
4. D. 16.
5. A. Matheran Light Railway

I2U2 Summit and Food Security

https://unacademy.com/course/mission-35-i2u2-juvenile-justice-amendment-act/4U9LSX01

1. Which of the following countries is not a part of I2U2?

A. India

B. Israel

C. Indonesia

D. UAE

2. I2U2 is also referred to as which of the following?

A. South Asian Quad

B. West Asian Quad

C. Mid Asian Quad

D. Asian Quad

105
3. Consider the following statements:

I. The Abraham Accord is the first Arab-Israeli peace deal in 26 years mediated by the UK.

II. I2U2 was initially formed in October, 2021

Which of the above statements is correct?

A. Only I

B. Only II

C. Both I & II

D. Neither I nor II

4. The Abraham Accord has been signed between Israel, the United Arab Emirates and which of
the following countries?

A. Saudi Arabia

B. Jordan

C. Kuwait

D. Bahrain

5. Consider the following statements:

I. The United Arab Emirates (UAE) announced plans to invest USD 2 billion in India to develop
Food Parks across the country.

II. Food Park is a concept that aims at establishing direct linkages from the farm to processing to
consumer markets.

Which of the above statements is incorrect?

A. Only I

B. Only II

106
C. Both I & II

D. Neither I nor II

Answer Key

1. C

2. B

3. B

4. D

5. D

Issue with the Juvenile Justice Amendment Act, 2021

https://unacademy.com/course/mission-35-i2u2-juvenile-justice-amendment-act/4U9LSX01

1. Consider the following statements:


I. Specifically, the amendment under challenge is the one to Section 86 of the JJ Act, which
provides that crimes against children which allow an imprisonment between three and seven years
will be deemed “cognisable”.
II. Parliament introduced and passed the Juvenile Justice (Care and Protection of Children) Act in
2015 to replace the Juvenile Delinquency Law and the Juvenile Justice (Care and Protection of
Children Act) 2000.
Choose the correct option:
a. I only
b. II only
c. Both I and II
d. Neither I nor II

2. Consider the following statements:

107
I. A non-cognizable offense is an offense in which the police officer as per the first schedule or
under any other law for the time being in force, can arrest the convict without a warrant and can
start an investigation without the permission of the court.
II. The first information report (FIR) is registered only in non-cognizable crimes.
Choose the correct option:
a. I only
b. II only
c. Both I and II
d. Neither I nor II

3. The Protection of Children from Sexual Offences Act (POCSO) was enacted in which year?
a. 2000
b. 2014
c. 2012
d. 2007

4. The National Crime Records Bureau was set up in 1986 under which Ministry?
a. Ministry of Defense
b. Ministry of Women and Child Development
c. Ministry of Home Affairs
d. None of the above

5. Consider the following statements:


I. Hague Convention on the Civil Aspects of International Child Abduction is an international
treaty to ensure the prompt return of the child who has been “abducted” from the country of their
“habitual residence”.
II. The Convention applies to children under the age of 18 years.
Choose the correct option:
a. I only
b. II only
c. Both I and II

108
d. Neither I nor II

1. B. II only.
2. D. Neither I nor II
3. C. 2012
4. C. Ministry of Home Affairs
5. A. I only.

National Initiative for Promoting Upskilling of Nirman Workers (NIPUN)

https://unacademy.com/course/mission-35-nipun-vpn-rules/JYSXV1K3

1. Consider the following statements:


I. Recently, an innovative project for skill training of construction workers called ‘NIPUN’ i.e.
The National Initiative for Promoting and Upgrading of Nirman Workers was launched.
II. NIPUN is creating a future labor force for the construction industry which will propel
innovation and large-scale development in the country.
Choose the correct option:
a. I only
b. II only
c. Both I and II
d. Neither I nor II

2. ___________ will be responsible for the overall execution of training, monitoring and candidate
tracking. Fill in the blank:
a. NITI Aayog
b. Ministry of Skill Development and Training
c. National Skill Development Corporation
d. Ministry of Housing and Urban Affairs

109
3. Consider the following statements:
I. The basic motive of the NIPUN project is to train over 1 lakh construction workers, through
fresh skilling and upskilling programmes.
II. The project NIPUN is an initiative of the Ministry of Skill Development and Training
Choose the correct option:
a. I only
b. II only
c. Both I and II
d. Neither I nor II

4. Consider the following statements:


I. Deendayal Antyodaya Yojana - National Livelihoods Mission (DAY-NULM) was launched by
the Ministry of Skill Development and Training, Government of India in 2011.
II. The Mission aims at creating efficient and effective institutional platforms for the rural poor
enabling them to increase household income through sustainable livelihood and improved access
to financial services.
Choose the correct option:
a. I only
b. II only
c. Both I and II
d. Neither I nor II

5. The Ministry of Education launched a National Initiative for Proficiency in Reading with
Understanding and Numeracy (NIPUN Bharat) in which year?
a. 2014
b. 2017
c. 2019
d. 2021

1. B. II only.
2. C. National Skill Development Corporation.

110
3. A. I only.
4. B. II only.
5. D. 2021.

New VPN Rules

https://unacademy.com/course/mission-35-nipun-vpn-rules/JYSXV1K3

1. Consider the following statements:


I. Recently, Indian Computer Emergency Response Team (CERT-In) issued norms under which
VPN providers have to record personal information of their customers, including the purpose of
using the service, for five years.
II. VPN stands for "Virtual Personal Network" and describes the opportunity to establish a
protected network connection when using public networks.
Choose the correct option:
a. I only
b. II only
c. Both I and II
d. Neither I nor II

2. The ____________ _ ministry issued new norms for VPN companies to record personal
information of their users including names, email id, phone number and IP address for a period of
five years. Fill in the blanks:
a. Ministry of Technology and Science
b. Ministry of Communication
c. Ministry of Electronics and Information Technology
d. Ministry of Information and Broadcasting

3. Consider the following statements:

111
I. A VPN hides the user’s IP address by letting the network redirect it through a specially
configured remote server run by a VPN host. This means that if a user is surfing online with a
VPN, the VPN server becomes the source of data.
II. Internet Service Providers (ISP) and other third parties can still see which websites the user
visits or data sent and received online.
Choose the correct option:
a. I only
b. II only
c. Both I and II
d. Neither I nor II

4. Consider the following statements:


I. An address that uniquely defines a hardware interface is called IP Address.
II. IP address changes each time a device is connected to the network as it is dynamically allocated
to the device when it participates in the network.
Choose the correct option:
a. I only
b. II only
c. Both I and II
d. Neither I nor II

5. Which Section of the Information Technology Act empowers CERT-IN to collect, analyze and
disseminate information on cyber security incidents?
a. Section 36A
b. Section 74
c. Section 70B
d. Section 76

1. A. I only.
2. C. Ministry of Electronics and Information Technology.
3. A. I only.

112
4. B. II only
5. C. Section 70B

CAATSA

https://unacademy.com/course/mission-35-caatsa-indo-aus-critical-mineral-
partnership/H5DSBEWX

1.Which country has designed the S-400 Triumf surface-to-air missile system?

(a) India

(b) China

(c) USA

(d) Russia

2.Which of the following statements is/are true?

I.The United States (US) House of Representatives has approved an amendment to the National
Defence Authorization Act (NDAA), proposing India-specific waiver under the Countering
America’s Adversaries Through Sanctions Act (CAATSA).

II.The National Defense Authorization Act (NDAA) is legislation that Congress passes each year
to make changes to the policies and organization of United States defense agencies and provide
guidance on how military funding can be spent.

(a) I only

(b) II only

(c) Both I and II

(d) Neither I nor II

3.Which of the following statements is/are true about the US’ CAATSA?

113
I. Its “ultimate goal”, “is to prevent revenue from flowing to the Chinese Government.

II.The CAATSA contains 12 types of sanctions. There are only two sanctions that may impact
either India-Russia relations or India-US relations.

(a) I only

(b) II only

(c) Both I and II

(d) Neither I nor II

4.In which year CAATSA came into effect?

(a) 2017

(b) 2018

(c) 2019

(d) 2020

5.Which company has designed and manufactured THAAD systems?

(a) General Dynamics

(b) Raytheon

(c) Lockheed Martin.

(d) Airbus Group

1.Answer:(d) Russia

2.Answer:(c) Both I and II

3.Answer:(b) II only

4.Answer:(a) 2017

114
5.Answer:(c) Lockheed Martin.

India-Australia Critical Minerals Investment Partnership

https://unacademy.com/course/mission-35-caatsa-indo-aus-critical-mineral-
partnership/H5DSBEWX
1.Which country is the world’s largest producer of critical minerals?

(a) China

(b) Australia

(c) Russia

(d) USA

2.Which of the following statements is/are true?

I.Critical minerals are elements that are the building blocks of essential modern-day technologies
and are at risk of supply chain disruptions.

II.India has set up KABIL or the Khanij Bidesh India Limited, a joint venture of three public sector
companies, to ensure a consistent supply of critical and strategic minerals to the Indian domestic
market.

(a) I only

(b) II only

(c) Both I and II

(d) Neither I nor II

3.Which of the following statements is/are true about FTAs?

I.The concept of free trade is in favor of trade protectionism or economic isolationism.

115
II.FTAs can be categorized as Preferential Trade Agreement, Comprehensive Economic
Cooperation Agreement (CECA), and Comprehensive Economic Partnership Agreement (CEPA).

(a) I only

(b) II only

(c) Both I and II

(d) Neither I nor II

4.Which country is not a Free trade partner of ASEAN?

(a) China

(b) India

(c) Japan

(d) Russia

5.What is the full form of FAME-II?

(a) Faster Adoption and Manufacturing of Electric Vehicles Scheme-II

(b) Faster Adoption and Manufacturing of Electronic Vehicles Scheme-II

(c) Faster Adoption and Manufacturing E-Vehicles Scheme-II

(d) Faster Adoption and Manufacturing Economic Scheme-II

(Answers)

1.(A)

2.(C)

3.(B)

4.(D)

116
5.(A)

NIRF Rankings 2022

https://unacademy.com/course/mission-35-caatsa-nirf-rankings/P2C6Y17W

1.Which of the following is not an Assessment Parameter of NIRF?

(a) Teaching, Learning and Resources

(b) Graduation Outcomes

(c) Outreach and Inclusivity

(d) Citations per faculty

2.Which of the following statements is/are true?

I. The National Institutional Ranking Framework (NIRF) was approved by the Ministry of
Education in September 2015.

II.Participation in NIRF is not compulsory for all government-run educational institutions.

(a) I only

(b) II only

(c) Both I and II

(d) Neither I nor II

3.Which of the following statements is/are true?

I. Quacquarelli Symonds (QS) is a leading global career and education network for ambitious
professionals looking to further their personal and professional development.

II.The ‘QS World University Rankings’ is an annual publication of university rankings which
comprises the global overall and subject rankings.

117
(a) I only

(b) II only

(c) Both I and II

(d) Neither I nor II

4.Which of the following is the Indian Initiative to improve the Indian education system?

(a) National Education Policy, 2020

(b) Impacting Research Innovation and Technology

(c) Institutions of Eminence (IoE) Scheme

(d) All of the Above

5.Which of the following pair is wrongly matched considering NIRF Rankings 2022?

(a) University: IISc, Bengaluru tops the category.

(b) Law: National Law School of India University, Bengaluru.

(c) Management: IIM Bengaluru.

(d) Medical: All India Institute of Medical Sciences, New Delhi

1.Answer:(d) Citations per faculty

2.Answer:(a) I only

3.Answer:(c) Both I and II

4.Answer:(d) All of the Above

5.Answer:(c) Management: IIM Bengaluru

118
World Population Prospects 2022

https://unacademy.com/course/mission-35-world-population-prospects-african-
union/SX6UCVLI

1. According to the 27th edition of the United Nations’ World Population Prospects (WPP), India
is projected to surpass China as the world’s most populous country by which year?
A. 2023
B. 2024
C. 2025
D. 2030

2. World Population Day is celebrated every year on which of the following days?
A. 10th July
B. 11th July
C. 12th July
D. 13th July

3. When did the United Nations publish the World Population Prospects for the first time?
A. 1950
B. 1951
C. 1952
D. 1953

4. Consider the following statements in relation to the significance of United Nations's Sustainable
Development Goals (SDGs).
I. Sustainable Development means development which meets the needs of the present without
compromising the ability of future generations to meet their own needs.
II. This most widely accepted definition of Sustainable Development was given by the Rotterdam
Commission in its report Our Common Future.
Which of the above statements is/are incorrect?
A. Only I

119
B. Only II
C. Both I and II
D. Neither I and II

5. Consider the following statements in relation to the significance of World Population Day on
July 11 every year.
I. World Population Day was established by the Governing Council of the United Nations
Development Programme in 1989.
II. It was inspired by the public interest in Five Billion Day, the approximate date on which the
world's population reached five billion people on July 11, 1987.
Which of the above statements is/are incorrect?
A. Only I
B. Only II
C. Both I and II
D. Neither I and II

Answer:
1. A
2. B
3. B
4. B
5. D

The African Union at 20

https://unacademy.com/course/mission-35-world-population-prospects-african-
union/SX6UCVLI

1. The African Union is celebrating its 20th Anniversary on which of the following dates?

A. 9 July 2022

B. 10 July 2022

120
C. 12 July 2022

D. 14 July 2022

2. Consider the following statements regarding the African Union.

I. The African Union (AU) is a continental body consisting of the 65 member states that make up
the countries of the African Continent.

II. In 2005, the Organization of African Unity was succeeded by the African Union, which had as
one of its goals to accelerate the "economic integration of the continent”

Which of the above statements is correct?

A. Only I

B. Only II

C. Both I & II

D. Neither I nor II

3. The ACFTA agreement was adopted and opened for signature on 21st March 2018 at which of
the following places?

A. Kigali

B. Dublin

C. Cape Town

D. Cairo

4. The African Continental Free Trade Area was established in which year?

A. 2017

B. 2018

C. 2019

121
D. 2020

5. Consider the following statements regarding the Indian Vice President's visit to Senegal in 2022.

I. Three MoUs (Memorandums of Understanding) for cultural exchange, cooperation in youth


matters and visa free regime were signed.

II. India offered a Special ITEC Course on the English training for Senegalese public servants for
20 persons at a time.

Which of the above statements is correct?

A. Only I

B. Only II

C. Both I & II

D. Neither I nor II

Answer Key

1. A

2. D

3. A

4. B

5. C

Sri Lanka Crisis

https://unacademy.com/course/mission-35-sl-crisisregistered-unrecognised-political-
parties/N4GYVG8J

122
1. In which year, all fertilizer imports were completely banned in Sri Lanka making it a 100%
organic farming nation?
A. 2019
B. 2020
C. 2021
D. 2022

2. India has started which of the following policies to bond with its neighbors to help Sri Lanka
out of the current crisis?
A. Neighborhood Policy
B. Neighbors' First Policy
C. Neighborhood First policy
D. None of the above

3. Which Sri Lankan President has recently resigned in light of anti-government protests in Sri
Lanka signaling political stability?
A. Ranil Wickremesinghe
B. Gotabaya Rajapaksa
C. Maithripala Sirisena
D. Mahinda Rajapaksa

4. Consider the following statements in relation to the working of Balance of Payments in a


country.
I. Balance of Payment (BoP) of a country can be defined as a systematic statement of all economic
transactions of a country with the rest of the world during a specific period, usually one year.
II. When exports exceed imports, there is a trade deficit and when imports exceed exports there is
a trade surplus.
Which of the above statements is/are correct?
A. Only I
B. Only II
C. Both I and II

123
D. Neither I and II

5. Consider the following statements in relation to the significance of Special Drawing Rights
(SDR).
I. The SDR is an international reserve asset, created by the IMF in 1969 to supplement its member
countries’ official reserves.
II. The SDR is an international reserve asset, created by the IMF in 1969 to supplement its member
countries’ official reserves.
Which of the above statements is/are incorrect?
A. Only I
B. Only II
C. Both I and II
D. Neither I and II

Answer:
1. C
2. C
3. B
4. A
5. D

Registered Unrecognized Political Parties (RUPP)

https://unacademy.com/course/mission-35-sl-crisisregistered-unrecognised-political-
parties/N4GYVG8J

1. Which of the following ordered the deletion of 111 registered unrecognized political parties that
it found to be “non-existent” and referred three of the parties to the Department of Revenue for
legal action for “serious financial impropriety"?

A. Supreme Court of India

124
B. Election Commission

C. Vigilance Commission of India

D. None of the above

2. Consider the following statements regarding a Recognised Political Party.

I. A recognised political party shall either be a National party or a State party if it meets certain
laid down conditions.

II. These parties are either newly registered parties or those which have not secured enough
percentage of votes in the assembly or general elections to become a state party, or those which
have never contested elections since being registered.

Which of the above statements is correct?

A. Only I

B. Only II

C. Both I & II

D. Neither I nor II

3. Which article of the constitution provides for the establishment of the Election Commission of
India?

A. Article 320

B. Article 324

C. Article 246

D. Article 326

4. Which of the following are conditions required for Recognition of National Parties?

125
I. If it secures 6% of valid votes polled in any four or more states at a general election to the Lok
Sabha or to the legislative assembly and in addition, it wins four seats in the Lok Sabha from any
state or states.

II. If it secures 6% of the valid votes polled in the state at a general election to the Lok Sabha from
the state concerned and in addition, it wins 1 seat in the Lok Sabha from the state concerned.

Which of the above statements is correct?

A. Only I

B. Only II

C. Both I & II

D. Neither I nor II

5. The Election Commission was established in accordance with the Constitution, in which of the
following year?

A. 1948

B. 1949

C. 1950

D. 1952

Answer Key

1. B

2. A

3. B

4. A

5. C

126
National Legal Services Authority

https://unacademy.com/course/mission-35-president-of-india-nalsa/NWE0STOW

1. Which of the following Articles of Indian Constitution provides that the State shall provide free
legal aid for promoting justice based on equal opportunity?
A. Article 39
B. Article 39A
C. Article 49
D. Article 49A

2. Consider the following statements in relation to the significance of establishment of the National
Legal Services Authority in India.
I. NALSA was founded in 1995 to monitor and review the effectiveness of legal aid programs and
to develop rules and principles for providing legal services under the Act.
II. It is governed under the Legal Services Authorities Act, 1985.
Which of the above statements is/are incorrect?
A. Only I
B. Only II
C. Both I and II
D. Neither I and II

3. Who is the patron in chief of National Legal Service Authority (NALSA) in India?
A. President
B. Chief Justice of Supreme Court
C. Prime Minister
D. Second senior most judge of Supreme Court

4. Consider the following statements in relation to the significance of the Designing Innovative
Solutions for Holistic Access to Justice (DISHA) scheme.

127
I. The Ministry of Electronics Information and Technology has launched a comprehensive,
holistic, integrated and systemic solution on access to justice at pan India level named as DISHA
scheme being implemented from 2021-26.
II. It aims to facilitate the District Judiciary for assisting in reduction of pendency of decade-old
pending cases in the district court & subordinate judiciary with the help of Nyaya Mitra.
Which of the above statements is/are incorrect?
A. Only I
B. Only II
C. Both I and II
D. Neither I and II

5. Which of the following persons are not eligible for legal aid under the Legal Services Authority
Act, 1987?
A. Women and Children
B. Persons in Custody
C. Members of SC/ST
D. None of the above

Answer:
1. B
2. B
3. B
4. A
5. D

Monkeypox

https://unacademy.com/course/mission-35-monkey-pox-beti-bachao-beti-padhao-
scheme/WNWBMHOM

1.Zoonotic transmission takes place between?

128
(a) Animal and Animal

(b) Animal and Man

(c) Man and Plants

(d) Man and Man

2.Which of the following statements is/are true?

I. Monkeypox is a viral zoonotic disease with symptoms similar to smallpox, although with less
clinical severity.The infection was first discovered in 1958 following two outbreaks of a pox-like
disease in colonies of monkeys kept for research — which led to the name ‘monkeypox’.

II.There is no specific treatment or vaccine available for Monkeypox infection.

(a) I only

(b) II only

(c) Both I and II

(d) Neither I nor II

3.Which of the following statements is/are true?

I. World Health Organization (WHO), the United Nations’ specialized agency for Health was
founded in 1948.It began functioning on April 7, 1948 – a date now being celebrated every year
as World Health Day

II.Regional office of WHO for South East Asia is located in New Delhi.

(a) I only

(b) II only

(c) Both I and II

(d) Neither I nor II

129
4.What is the full form of RT-PCR?

(a) Reverse transcriptase polymerase chain reaction

(b) Reverse transcription polymer chain reaction

(c) Reverse transcriptase polymer chain reaction

(d) Reverse transcrip polymerise chain reaction

5. Which of the following diseases is transmitted primarily by Mosquitoes?

(a) Zika virus

(b) Yellow Fever

(c) Dengue

(d) All of the Above

1.Answer:(b) Animal and Man

2.Answer:(c) Both I and II

3.Answer:(c) Both I and II

4.Answer:(a) Reverse transcriptase polymerase chain reaction

5.Answer:(d) All of the Above

Beti Bachao Beti Padhao Scheme

https://unacademy.com/course/mission-35-monkey-pox-beti-bachao-beti-padhao-
scheme/WNWBMHOM

130
1. The Government aims to increase girls’ participation in sports by identifying talent and linking
them with appropriate authorities under which of the following scheme?

A. Khelo India

B. Sab Padhe, Sab Badhe

C. Ab Khelega India

D. Game Dikha India

2. The Beti Bachao Beti Padhao (BBBP) Scheme was launched in which year?

A. 2012

B. 2013

C. 2015

D. 2014

3. Consider the following statements:

I. In 2021 parliamentary committee on empowerment of women in its report stated that nearly 80
% funds for the BBBP scheme has been used for advertising, and not on sectoral interventions
such as women’s health and education.

II. BBBP is a joint initiative of the Ministry of Women and Child Development, Ministry of Health
and Family Welfare and Ministry of Human Resource Development.

Which of the above statement is incorrect?

A. Only I

B. Only II

C. Both I & II

D. Neither I nor II

131
4. Consider the following statements:

I. India celebrates National Girl Child Day every year on 25th January.

II. The National Girl Child Day was first initiated in 2008 by the Ministry of Women and Child
Development.

Which of the above statement is correct?

A. Only I

B. Only II

C. Both I & II

D. Neither I nor II

5. The PC-PNDT (Pre-Conception & Pre-Natal Diagnostic Techniques) Act was passed in which
year?

A. 1992

B. 1993

C. 1994

D. 1995

Answer Key

1. A

2. C

3. D

4. B

5. A

132
Hi,

NerdDose was started with the mission of making CLAT preparation Simple ‘nd
Sensible. Our class at 10.30 PM has simplified Current Affairs for a lot of CLAT
aspirants. We remain indebted to everyone who has supported us in this journey
and helped us become one of the most viewed CLAT Current Affairs class of the
country. We ask you for nothing. No subscriptions, no likes, no shares. Just one
thing – Join the #LetsCLATit Mission. LetsCLATit together! Only 90 days to go
and we trust we will do it, together.

We have recently started uploading a lot of free content on our Youtube Channel –
https://www.youtube.com/nerddoseclat

We hope that you enjoy the content that we upload there and keep reaching out to
us on nerddose@gmail.com for any suggestions, complaints or questions.

133

You might also like